File Download Area

Information about "Official SAT Practice Test 2004-2005.pdf"

  • Filesize: 2.49 MB
  • Uploaded: 14/12/2018 20:38:26
  • Status: Active

Free Educational Files Storage. Upload, share and manage your files for free. Upload your spreadsheets, documents, presentations, pdfs, archives and more. Keep them forever on this site, just simply drag and drop your files to begin uploading.

Download Urls

  • File Page Link
    https://www.edufileshare.com/b1cbbfe809faa214/Official_SAT_Practice_Test_2004-2005.pdf
  • HTML Code
    <a href="https://www.edufileshare.com/b1cbbfe809faa214/Official_SAT_Practice_Test_2004-2005.pdf" target="_blank" title="Download from edufileshare.com">Download Official SAT Practice Test 2004-2005.pdf from edufileshare.com</a>
  • Forum Code
    [url]https://www.edufileshare.com/b1cbbfe809faa214/Official_SAT_Practice_Test_2004-2005.pdf[/url]

[PDF] Official SAT Practice Test 2004-2005.pdf | Plain Text

SAT Preparation Booklet ™ 2004–2005 For the new SAT ® Visit the SAT Preparation Center at www.collegeboard.com for more practice

SAT Preparation Booklet 2 The College Board: Connecting Students to College Success The College Board is a not-for-profit membership associa- tion whose mission is to connect students to college suc- cess and opportunity. Founded in 1900, the association is composed of more than 4,500 schools, colleges, universi- ties, and other educational organizations. Each year, the College Board serves over three million students and their parents, 23,000 high schools, and 3,500 colleges through major programs and services in college admissions, guid- ance, assessment, financial aid, enrollment, and teaching and learning. Among its best-known programs are the SAT®, the PSAT/NMSQT®, and the Advanced Placement Program® (AP®). The College Board is committed to the principles of excellence and equity, and that commitment is embodied in all of its programs, services, activities, and concerns. For further information, visit www.collegeboard.com. Copyright © 2004 by College Entrance Examination Board. All rights reserved. Advanced Placement Program, AP, College Board, SAT, and the acorn logo are registered trademarks of the College Entrance Examination Board. Connect to college success, SAT Reasoning Test, SAT Subject Test, The Official Guide to the SAT: For the New SAT, and The Official SAT Online Course are trademarks owned by the College Entrance Examination Board. PSAT/NMSQT is a registered trademark of the College Entrance Examination Board and National Merit Scholarship Corporation. Contents The Critical Reading Section . . . . . . . . . . . . . . . .6 Sentence Completions . . . . . . . . . . . . . . . . . . . . . . . . . . . . .6 Passage-based Reading . . . . . . . . . . . . . . . . . . . . . . . . . . . . .7 The Math Section . . . . . . . . . . . . . . . . . . . . . . . .14 Calculator Policy . . . . . . . . . . . . . . . . . . . . . . . . . . . . . . . . .14 Math Review . . . . . . . . . . . . . . . . . . . . . . . . . . . . . . . . . . . .15 Multiple-Choice Questions . . . . . . . . . . . . . . . . . . . . . . . . .21 Student-Produced Response . . . . . . . . . . . . . . . . . . . . . . . .24 The Writing Section . . . . . . . . . . . . . . . . . . . . . .27 Improving Sentences . . . . . . . . . . . . . . . . . . . . . . . . . . . . . .27 Identifying Sentence Errors . . . . . . . . . . . . . . . . . . . . . . . .28 Improving Paragraphs . . . . . . . . . . . . . . . . . . . . . . . . . . . .29 The Essay . . . . . . . . . . . . . . . . . . . . . . . . . . . . . . . . . . . . . . .31 Scoring the Essay . . . . . . . . . . . . . . . . . . . . . . . . . . . . . . . . .34 The Practice SAT . . . . . . . . . . . . . . . . . . . . . . . . .36 About the Practice Test . . . . . . . . . . . . . . . . . . . . . . . . . . . .36 Answer Sheet . . . . . . . . . . . . . . . . . . . . . . . . . . . . . . . . . . . .37 Official Practice Test . . . . . . . . . . . . . . . . . . . . . . . . . . . . . .45 Answer Key . . . . . . . . . . . . . . . . . . . . . . . . . . . . . . . . . . . . .83 Scoring the Practice Test . . . . . . . . . . . . . . . . . . . . . . . . . . .84 Score Conversion Table . . . . . . . . . . . . . . . . . . . . . . . . . . . .85 Test Development Committees . . . . . . . . . . . . .87

SAT Preparation Booklet 3 SAT Reasoning Test ™ This booklet will answer your questions about and help you practice for the new SAT. SAT FAQs Why should I take the SAT? The SAT Reasoning Test is a measure of the critical think- ing skills you will need for academic success in college. The SAT assesses how well you analyze and solve problems— skills you learned in school that you will need in college. What is the new SAT? Starting in March 2005, you will be taking the new SAT. The SAT is changing to keep up-to-date with current teaching practices and high school and college curricula. The three sections of the new SAT are: ● Critical Reading, which has sentence completion and passage-based questions ● Math, which is based on the math that college- bound students typically learn during their first three years of high school ● Writing, which has multiple-choice questions and a written essay What do I need to know about the essay? Colleges want to know not only how well you write, but also how well you express and then back up a point of view. You will have 25 minutes to write your essay, which will count for approximately 30% of the score for the writ- ing section. The essay will be scored as a first draft, not as a polished piece of writing. How can I best prepare for the SAT? ● Take the PSAT/NMSQT®. This test is given in high schools in October. Many students take it during their sophomore and junior years. ● Review the sample questions, test-taking sugges- tions, and directions in this booklet. ● Take the official practice SAT in this booklet. ● Visit the SAT Preparation Center at www.collegeboard.com. How should I get ready for test day? ● Make sure you have on hand all the materials you will need, such as a calculator, your admission ticket, and an official photo ID. ● Check out the route to the test center and know where the weekend entrances are located. ● Get a good night’s sleep. ● Leave yourself plenty of time so you’ll arrive at the test center a little early. What are the important points to know about the SAT? ● Get to know as much about the test as you can. ● Become familiar with the test directions. ● Become familiar with the answer sheet on pages 37–43. ● Know what to expect from the test, the types of questions, and the numbers of questions. ● Learn how to approach each type of question. How can I help myself feel as confident as possible? ● Think positively. ● Stay focused. ● Concentrate only on what you are doing. ● Keep the test in perspective. ● Remember that you are in control. Approaches to taking the SAT ● Answer easy questions first.The easier questions are at the start of the section and the harder ones are at the end. The exception is in the critical read- ing section, where questions are ordered according to the logic and organization of each passage. ● Make educated guesses.Ifyou can rule out one or more answer choices for multiple-choice questions, you have a better chance of guessing the right answer. ● Skip questions that you really can’t answer.You can get an average score by answering about half of the questions correctly and skipping the remaining questions. ● Keep track of time.Don’t spend too much time on any one section. ● Use your test booklet as scratch paper. ● Mark questions you skipped and want to return to. ● Check your answer sheet to make sure you are answering the right question. ●You have 3 hours and 45 minutes to complete the entire test. ●All multiple-choice questions are scored the same: one point for each correct answer, and one quarter point subtracted for a wrong answer. ●The SAT is one factor colleges look at when they consider your application. ●You can always take the test again. One out of every two high school students takes the SAT at least twice.

Plus, get essay scores immediately • Practice for the new SAT essay by responding to essay questions and getting back computer-generated estimated scores. • Receive estimated scores for: • The practice essay question in this booklet • The essay question included with this booklet’s full-length practice test • All essay questions on the online course • All essay questions in The Offi cial SAT Study Guide: For the New SA T Subscribe Now! collegeboard.com/s atonlinecourse • 18 interactive lessons cover the new SAT ® math, reading, and writing sections • 3 practice tests and 600+ practice questions— all follow proprietary new SAT specifi cations • Detailed personalized feedback on all practice tests and quizzes • Explanations of answers to all questions on the online course and to all 8 practice tests in The Offi cial SAT Study Guide: For the New SA T™ The Of fi cial SAT Online Course ™ Prepare for the new SAT ® with help from the test maker. A personalized, comprehensive online course featuring:

The Of fi cial SAT Study Guide: For the New SA T ™ Prepare with t he only book of new SAT ® practice tests created by the test maker. This comprehensive study guide provides 800+ pages and 21 chapters packed with 8 full-length practice tests and hundreds of additional questions— all written by the makers of the SAT. Available at collegeboard.com and bookstores everywhere. • Receive FREE score and skills reports online, plus sample essays for the book’s 8 practice tests. • Receive a $10 DISCOUNT on The Offi cial SAT Online Course ™, which includes auto essay scoring and answer explanations for the book’s 8 practice tests.

The Critical Reading Section The critical reading section of the SAT contains two types of questions: ● sentence completions (19 questions) ● passage-based reading (48 questions) Note:Calculators may not be on your desk or be used during the critical reading section of the SAT. Approaches to the Critical Reading Section ● Work on sentence completion questions first. They take less time to answer than the passage-based reading questions. ● In your test booklet, mark each question you don’t answer so that you can easily go back to it later if you have time. ● Do not jump from passage to passage. Stay with a passage until you have answered as many questions as you can. ● The difficulty of sentence completion questions increases as you move through a question set. ● Reading questions do not increase in difficulty from easy to hard. Instead, they follow the logic of the passage. ● When you have gone through all the questions associated with a passage, go back and review any you left out or weren’t sure about. Remember that all questions are worth the same number of points regardless of the type or difficulty. ● The information you need to answer each reading question is always in the passage(s). Careful read- ing is the key to finding the correct answer. Don’t be misled by an answer that looks correct but is not supported by the actual text. ● Ifyou don’t know what a word means in a sentence completion or reading passage, consider related words, familiar sayings and phrases, roots, prefixes, and suffixes. Have you ever heard or seen a word that may be related to it? Sentence Completions Sentence completion questions measure your: ● knowledge of the meanings of words ● ability to understand how the different parts of a sentence fit logically together Directions Each sentence below has one or two blanks, each blank indicating that something has been omitted. Beneath the sentence are five words or sets of words labeled A through E. Choose the word or set of words that, when inserted in the sentence, b est fits the meaning of the sentence as a whole. 1. Hoping to ------- the dispute, negotiators proposed a compromise that they felt would be ------- to both labor and management. (A) enforce . . useful (B) end . . divisive (C) overcome . . unattractive (D) extend . . satisfactory (E) resolve . . acceptable Answering Sentence Completion Questions One way to answer a sentence completion question with two words missing is to focus first on just one of the two blanks. If one of the words in an answer choice is logically wrong, then you can eliminate the entire choice from consideration. ● Look at the first blank in the example above. Would it make sense to say that “negotiators” who have “proposed a compromise” were hoping to enforceor extendthe “dispute”? No, so neither (A) nor (D) can be the correct answer. ● Now you can focus on the second blank. Would the “negotiators” have proposed a compromise that they believed would be divisiveor unattractiveto “both labor and management”? No, so (B) and (C) can be eliminated, and only choice (E) remains. ● Always check your answer by reading the entire sentence with your choice filled in. Does it make sense to say “Hoping to resolvethe dispute, the negotiators proposed a compromise that they felt would be acceptableto both labor and manage- ment”? Yes. Correct answer: (E) / Difficulty level: Easy SAT Preparation Booklet 6

SAT Preparation Booklet 7 2. Because King Philip’s desire to make Spain the dominant power in sixteenth-century Europe ran counter to Queen Elizabeth’s insistence on autonomy for England, ------- was -------. (A) reconciliation . . assured (B) warfare . . avoidable (C) ruination . . impossible (D) conflict . . inevitable (E) diplomacy . . simple Be sure to look for key words and phrases as you read each sentence. Words such as although,however,if,but,and since are important to notice because they signal how the differ- ent parts of a sentence are logically related to each other. Wo rds such as notand neverare important because they indicate negation. In the example above, the entire sen- tence hinges on a few key words: “Becausesomething ran counter tosomething else,blankwas blank.” ● The word “because” indicates that the information in the first part of the sentence (the part before the comma) explains the reason for the situation described in the second part. The first part states that what King Philip wanted (domination for Spain) “ran counter to” what Queen Elizabeth wanted (independence for England). ● Given that there was such a fundamental disagree- ment between the two monarchs, would reconcili- ationbe assured? Unlikely. ● Would warfarebe avoidable? Hardly; warfare might be unavoidable. ● Would ruinationbe impossible? No. ● Would diplomacybe simple? Not necessarily. ● Only choice (D) fits logically with the key words in the sentence:Becausewhat one person wanted ran counter towhat another person wanted,conflict was inevitable. Correct answer: (D) / Difficulty level: Medium 3. There is no doubt that Larry is a genuine -------: he excels at telling stories that fascinate his listeners. (A) braggart (B) dilettante (C) pilferer (D) prevaricator (E) raconteur Some sentence completions contain a colon. This is a signal that the words after the colon define or directly clarify what came before. In this case, “he excels at telling stories that fascinate his listeners” serves to define the word raconteur,choice (E). None of the other words is directly defined by this clause. ● A braggartmay or may not excel at telling stories and may actually annoy listeners. ● A dilettanteis someone who dabbles at a career or hobby and so may not excel at anything. ● A pilferersteals repeatedly, in small quantities; this has nothing to do with storytelling. ● A prevaricatortells lies, but not necessarily in an accomplished or fascinating way; and the sentence refers to stories, not lies. You should choose the word that best fits the meaning of the sentence as a whole, and only choice (E) does so. Correct answer: (E) / Difficulty level: Hard Passage-based Reading The reading questions on the SAT measure your ability to read and think carefully about several different passages ranging in length from about 100 to about 850 words. Passages are taken from a variety of fields, including the humanities, social studies, natural sciences, and literary fiction. They vary in style and can include narrative, argu- mentative, and expository elements. Some selections con- sist of a pair of related passages on a shared issue or theme that you are asked to compare and contrast. The following kinds of questions may be asked about a passage: ● Vocabulary in Context :These questions ask you to determine the meanings of words from their context in the reading passage. ● Literal Comprehension:These questions assess your understanding of significant information directly stated in the passage. ● Extended Reasoning:These questions measure your ability to synthesize and analyze information as well as to evaluate the assumptions made and the techniques used by the author. Most of the reading questions fall into this category. You may be asked to identify cause and effect, make infer- ences, recognize a main idea or an author’s tone, and follow the logic of an analogy or an argument. Answering Passage-based Reading Questions Below are samples of the kinds of reading passages and questions that may appear on your test. For each set of sample materials: ● read the passage carefully, ● decide on the best answer to each question, and then ● read the explanation for the correct answer.

Some of the reading passages in the SAT are as short as a paragraph or two, about 100 words in length. You will also find one or more pairsof related short passages in each edition of the test. Such material will be followed by two to six questions that measure the same kinds of reading skills as are measured by the questions following longer passages. Directions The passages below are followed by questions based on their content; questions following a pair of related passages may also be based on the relationship between the paired passages. Answer the questions on the basis of what is stated or implied in the passages and in any introductory material that may be provided. Questions 4-5 are based on the following passage. “The rock was still wet. The animal was glistening, like it was still swimming,” recalls Hou Xianguang. Hou discovered the unusual fossil while surveying rocks as a paleontology graduate student in 1984, near the Chinese town of Chengjiang. “My teach- ers always talked about the Burgess Shale animals. It looked like one of them. My hands began to shake.” Hou had indeed found a Naraoia like those from Canada. However, Hou’s animal was 15 million years older than its Canadian relatives. Sample Questions Some questions ask you to recognize the meaning of a word as it is used in the context of the passage. 4. In line 4, “surveying” most nearly means (A) calculating the value of (B) examining comprehensively (C) determining the boundaries of (D) polling randomly (E) conducting a statistical study of The word “surveying” has a number of meanings, several of which are included in the choices above. In the context of this passage, however, only (B) makes sense. A student in the field of “paleontology” is one who studies prehistoric life as recorded in fossil remains. One of the activities of such a student would be to examine rocks carefully and “comprehensively” while looking for fossils. ● (A), (C), and (E) are incorrect because someone who studies fossils would not calculate the “value” of rocks, or determine the “boundaries” of rocks, or conduct a “statistical study” of rocks. ● (D) is wrong because “polling” rocks makes no sense at all. Correct answer: (B) / Difficulty level: EasyYou may be asked to make an inference or draw a conclusion about a statement made in the passage. 5. It can be inferred that Hou Xianguang’s “hands began to shake” (line 9) because Xianguang was (A) afraid that he might lose the fossil (B) worried about the implications of his finding (C) concerned that he might not get credit for his work (D) uncertain about the authenticity of the fossil (E) excited about the magnitude of his discovery In the passage, Hou states that the fossil that he found “looked like” certain other fossils that his “teacher always talked about.” He understands almost immediately, there- fore, the significance of what he has found, and so (E) is the correct answer: Hou’s hands were shaking because he was “excited about the magnitude of his discovery.” ● (A) is wrong because there is no suggestion that Hou was “afraid that he might lose the fossil.” ● (B) and (C) are wrong because Hou was not “wor- ried about” his discovery or “concerned that he might not get credit.” The passage indicates only that Hou recognized that he had found something valuable. ● (D) is wrong because Hou’s immediate reaction is that he thinks he has found an important fossil. The first two sentences of the passage dramatize the discovery; it is Hou’s excitement that causes him to tremble, not his uncertainty. Correct answer: (E) / Difficulty level: Easy Questions 6–9 are based on the following passage. This passage is adapted from a novel written by a woman in 1899. The novel was banned in many places because of its unconventional point of view. It was eleven o’clock that night when Mr. Pontellier returned from his night out. He was in an excellent humor, in high spir- its, and very talkative. His entrance awoke his wife, who was in bed and fast asleep when he came in. He talked to her while he undressed, telling her anecdotes and bits of news and gossip that he had gathered during the day. She was overcome with sleep, and answered him with little half utterances. He thought it very discouraging that his wife, who was the sole object of his existence, evinced so little interest in things which concerned him and valued so little his conversation. Mr. Pontellier had forgotten the bonbons and peanuts that he had promised the boys. Notwithstanding, he loved them very much and went into the adjoining room where they slept to take a look at them and make SAT Preparation Booklet 8 Line 5 10 Line 5 10 15 20

An indescribable oppression, which seemed to generate in some unfamiliar part of her consciousness, filled her whole being with a vague anguish. It was like a shadow, like a mist passing across her soul’s summer day. It was strange and unfamiliar; it was a mood. She did not sit there inwardly upbraiding her husband, lamenting at Fate, which had directed her footsteps to the path which they had taken. She was just having a good cry all to herself. The mosquitoes suc- ceeded in dispelling a mood which might have held her there in the darkness half a night longer. The following morning Mr. Pontellier was up in good time to take the carriage which was to convey him to the steamer at the wharf. He was returning to the city to his business, and they would not see him again at the Island till the coming Saturday. He had regained his composure, which seemed to have been somewhat impaired the night before. He was eager to be gone, as he looked forward to a lively week in the financial center. Sample Questions Following are four sample questions about this passage. In the actual test, as many as thirteen questions may appear with a passage of this length. You may be asked to interpret information presented throughout the passage and to evaluate the effect of the lan- guage used by the author. 6. The narrator would most likely describe Mr. Pontellier’s conduct during the evening as (A) typically generous (B) justifiably impatient (C) passionate and irrational (D) patronizing and self-centered (E) concerned and gentle This question asks you to consider a large portion of the passage and to make an inference about the narrator’s view of “Mr. Pontellier’s conduct during the evening.” To answer such a question, you should look carefully at the particular words used and details mentioned in the passage. In the first paragraph, Mr. Pontellier awakens his wife after his “night out”; he seems not to notice or care that she had been sound asleep. In lines 38-47, the narrator describes Mr. Pontellier speaking to his wife in a superior and con- descending manner about “a mother’s place” in caring for children and about how hard he works at “his brokerage business.” ● (A) and (E) are not correct because the narrator does not depict Mr. Pontellier’s words and actions during the evening as “generous” or “gentle.” SAT Preparation Booklet 9 sure that they were resting comfortably. The result of his investigation was far from satisfactory. He turned and shifted the youngsters about in bed. One of them began to kick and talk about a basket full of crabs. Mr. Pontellier returned to his wife with the information that Raoul had a high fever and needed looking after. Then he lit his cigar and went and sat near the open door to smoke it. Mrs. Pontellier was quite sure Raoul had no fever. He had gone to bed perfectly well, she said, and nothing had ailed him all day. Mr. Pontellier was too well acquainted with fever symptoms to be mistaken. He assured her the child was burning with fever at that moment in the next room. He reproached his wife with her inatten- tion, her habitual neglect of the children. If it was not a mother’s place to look after chil- dren, whose on earth was it? He himself had his hands full with his brokerage business. He could not be in two places at once; mak- ing a living for his family on the street, and staying home to see that no harm befell them. He talked in a monotonous, insistent way. Mrs. Pontellier sprang out of bed and went into the next room. She soon came back and sat on the edge of the bed, leaning her head down on the pillow. She said noth- ing, and refused to answer her husband when he questioned her. When his cigar was smoked out he went to bed, and in half a minute was fast asleep. Mrs. Pontellier was by that time thor- oughly awake. She began to cry a little, and wiped her eyes on the sleeve of her night- gown. She went out on the porch, where she sat down in the wicker chair and began to rock gently to and fro. It was then past midnight. The cottages were all dark. There was no sound abroad except the hooting of an old owl and the everlasting voice of the sea, that broke like a mournful lullaby upon the night. The tears came so fast to Mrs. Pontellier’s eyes that the damp sleeve of her nightgown no longer served to dry them. She went on crying there, not caring any longer to dry her face, her eyes, her arms. She could not have told why she was crying. Such experi- ences as the foregoing were not uncommon in her married life. They seemed never before to have weighed much against the abundance of her husband’s kindness and a uniform devotion which had come to be tacit and self-understood. 80 85 90 95 100 25 30 35 40 45 50 55 60 65 70 75

SAT Preparation Booklet 10 ● (B) is not correct because the narrator does not suggest that Mr. Pontellier’s conduct with his wife is justifiable. ● (C) is not correct; although Mr. Pontellier’s behavior is selfish and inconsiderate, it is not “passionate” —in fact, the narrator states that Mr. Pontellier “talked in a monotonous, insistent way.” ● (D) is correct because it accurately describes the narrator’s impression of Mr. Pontellier’s behavior during the evening, “patronizing and self-cen- tered.” Someone who is “patronizing” has an atti- tude of superiority and thus treats others as if they were less important. Correct answer: (D) / Difficulty level: Medium Some questions ask you to focus on a specific piece of infor- mation presented in the passage. 7. In context, the description in lines 46-47 of Mr. Pontellier’s way of speaking suggests the narrator’s belief that his complaints are (A) stumbling and confused (B) familiar and not as urgent as he claims (C) angry and sarcastic (D) too complex to make sense to anyone but himself (E) both rational and thought-provoking In lines 46-47, the narrator describes Mr. Pontellier’s “way of speaking” as “monotonous, insistent.” Previously, Mr. Pontellier had told his wife that one of their sons “had a high fever and needed looking after,” and he had criticized Mrs. Pontellier for her “habitual neglect of the children.” These are seemingly serious matters, and yet Mr. Pontellier is described as not at all excited in the way that he commu- nicates his opinions to his wife. ● (A) is wrong because Mr. Pontellier speaks assertively to his wife throughout the passage, not in a “stumbling” or uncertain manner. ● (C) is wrong because statements that are “monoto- nous” and “insistent” are not “angry and sarcastic.” ● (D) and (E) are wrong because the narrator does not indicate that Mr. Pontellier’s statements to his wife are “too complex to make sense” or “rational and thought-provoking.” In fact, the terms “monotonous” and “insistent” suggest that the statements are rather dull and simpleminded. ● The correct answer is (B) because concerns that are voiced “in a monotonous, insistent way” are likely to be ones that are oft-repeated and “familiar,” and probably “not as urgent” as Mr. Pontellier claims. The statement in lines 53-55 also supports this answer: “When his cigar was smoked out he went to bed, and in half a minute was fast asleep.” Correct answer: (B) / Difficulty level: HardSome questions require you to make an inference or draw a conclusion about what you have read. 8. In lines 56-92, Mrs. Pontellier’s reactions to her husband’s behavior on returning home suggest that (A) she accepts unquestioningly her role of caring for the children (B) this is one of the first times she has acknowledged her unhappiness (C) her marriage is not what is making her so depressed (D) she is angry about something that happened before her husband went out (E) she is not as worldly as her husband is In these lines, Mrs. Pontellier cries for a long time while sitting alone on the porch. Her husband’s treatment of her has upset her greatly. The narrator indicates that such behavior by Mr. Pontellier was “not uncommon” but that Mrs. Pontellier had not previously been too bothered by such incidents: “They seemed never before to have weighed much against the abundance of her husband’s kindness ....” ● (A) is not correct because the issue of “caring for the children” is not the focus of this part of the passage; Mrs. Pontellier’s feelings of sadness and “oppression” are not related to her acceptance of “her role” as a mother. ● (C) is not correct because it is precisely her rela- tionship with her husband that has made her “so depressed.” ● (D) is not correct because there is no indication in the passage that “something that happened before her husband went out” has made Mrs. Pontellier “angry.” In fact, it is his behavior aft erhis return that has upset her. ● (E) is not correct because whether Mrs. Pontellier is “as worldly as her husband” is irrelevant to her reaction to his treatment of her; the passage sug- gests not that she lacks sophistication but that he lacks consideration. ● (B) is correct because Mrs. Pontellier’s “strange and unfamiliar” mood of “oppression” and “anguish” marks a new realization on her part of her “unhap- piness” with her husband. Correct answer: (B) / Difficulty level: Medium You may be asked to consider the overall description of a character, event, or phenomenon across an entire passage. 9. The passage shows Mr. Pontellier as happiest when he (A) is attending to his children (B) sits outside and smokes a cigar (C) makes up with his wife after an argument (D) has been away from home or is about to leave home (E) has showered his children with gifts of candy

SAT Preparation Booklet 11 The passage begins with Mr. Pontellier “in an excellent humor,” having just returned after a night away from home. He becomes less happy, however, when his wife is too sleepy to talk with him, and when he discovers that his son Raoul “had a high fever and needed looking after.” Subsequently, he lectures his wife about their family roles and responsibilities, finishes his cigar, and goes to bed. The next morning, Mr. Pontellier has “regained his composure” and is “eager to be gone, as he looked forward to a lively week” away from his family at work. ● (A) and (E) are not correct because Mr. Pontellier gets upset the one time that he is “attending to” his sons, and he has forgotten to send them the treats that he had promised. ● (B) is not correct because Mr. Pontellier is described as neither happy nor unhappy while he smokes; there are other occasions in the passage when he is “happier.” ● (C) is not correct because the passage never shows Mr. Pontellier making up with his wife after their argument. ● (D) is the correct answer based on the description of a happy Mr. Pontellier at the beginning and the end of the passage, when “he has been away from home or is about to leave home.” Correct answer: (D) / Difficulty level: Medium Questions 10-13 are based on the following passages. These two passages were adapted from autobiographical works. In the first, a playwright describes his first visit to a theater in the 1930’s; in the second, an eighteenth-century writer describes two visits to theaters in London. Passage 1 I experienced a shock when I saw a cur- tain go up for the first time. My mother had taken me to see a play at the Schubert Theater on Lenox Avenue in Harlem in New York City.Here were living people talking to one another inside a large ship whose deck actually heaved up and down with the swells of the sea. By this time I had been going to the movies every Saturday afternoon —Charlie Chaplin’s little comedies, adven- ture serials, Westerns. Yet once you knew how they worked, movies, unlike the stage, left the mind’s grasp of reality intact since the happenings were not in the theater where you sat. But to see the deck of the ship in the theater moving up and down, and people appearing at the top of a ladder or disappearing through a door—where did they come from and where did they go? Obviously into and out of the real world of Lenox Avenue. This was alarming. And so I learned that there were two kinds of reality, but that the stage was farmore real. As the play’s melodramatic story developed, I began to feel anxious, for there was a villain on board who had a bomb and intended to blow everybody up. All over the stage people were looking for him but he appeared, furtive and silent, only when the searchers were facing the other way. They looked for him behind posts and boxes and on top of beams, even after the audience had seen him jump into a barrel and pull the lid over him. People were yelling, “He’s in the barrel,” but the passengers were deaf. What anguish! The bomb would go off any minute, and I kept clawing at my mother’s arm, at the same time glancing at the the- ater’s walls to make sure that the whole thing was not really real. The villain was finally caught, and we happily walked out onto sunny Lenox Avenue, saved again. Passage 2 I was six years old when I saw my first play at the Old Drury. Upon entering the theater, the first thing I beheld was the green curtain that veiled a heaven to my imagina- tion. What breathless anticipations I endured! I had seen something like it in an edition of Shakespeare, an illustration of the tent scene with Diomede in Troilus and Cressida.(A sight of that image can always bring back in a measure the feeling of that evening.) The balconies at that time, full of well-dressed men and women, projected over the orchestra pit; and the pilasters* reaching down were adorned with a glister- ing substance resembling sugar candy. The orchestra lights at length rose. Once the bell sounded. It was to ring out yet once again— and, incapable of the anticipation, I reposed my shut eyes in a sort of resignation upon my mother’s lap. It rang the second time. The curtain drew up—and the play was Artaxerxes! Here was the court of ancient Persia. I took no proper interest in the action going on, for I understood not its import. Instead, all my feeling was absorbed in vision. Gorgeous costumes, gardens, palaces, princesses, passed before me. It was all enchantment and a dream. After the intervention of six or seven years I again entered the doors of a theater. That old Artaxerxesevening had never done ringing in my fancy. I expected the same feelings to come again with the same occa- sion. But we differ from ourselves less at sixty and sixteen, than the latter does from six. In that interval what had I not lost! At six I knew nothing, understood nothing, discriminated nothing. I felt all, loved all, Line 5 10 15 2045 50 55 60 65 70 75 8025 30 35 40

wondered all. I could not tell how, but I had left the temple a devotee, and was returned a rationalist. The same things were there materially; but the emblem, the reference, was gone. The green curtain was no longer a veil, drawn between two worlds, the unfold- ing of which was to bring back past ages, but a certain quantity of green material, which was to separate the audience for a given time from certain of their fellows who were to come forward and pretend those parts. The lights—the orchestra lights—came up a clumsy machinery. The first ring, and the second ring, was now but a trick of the prompter’s bell. The actors were men and women painted. I thought the fault was in them; but it was in myself, and the alteration which those many centuries—those six short years—had wrought in me. * Pilasters are ornamental columns set into walls. Sample Questions Following are four sample questions about this pair of related passages. In the test, some questions will focus on Passage 1, others will focus on Passage 2, and about half or more of the questions following each pair of passages will focus on the relationships between the passages. Some questions require you to identify shared ideas or simi- larities between the two related passages. 10. The authors of both passages describe (A) a young person’s sense of wonder at first seeing a play (B) a young person’s desire to become a playwright (C)the similarities between plays and other art forms (D) how one’s perception of the theater may develop over time (E) the experience of reading a play and then seeing it performed To answer this question, you have to figure out what these two passages have in common. The subject of Passage 1 is a child’s first visit to see a play performed in a theater, and how captivated he was by the entire experience. Passage 2 describes two different visits to the theater; at age six the child is entranced by the spectacle of the performance but, “after the intervention of six or seven years,” the older and now more knowledgeable child is not so impressed.(A) is the correct answer because all of Passage 1 and the first half of Passage 2 describe “a young person’s sense of won- der at first seeing a play.” ● (B) is wrong; even though the introduction to these passages reveals that one of the authors is a “playwright,” there is no mention in either passage of a “desire to become a playwright.” ● (C) is wrong because Passage 1 mentions differ- ences rather than “similarities” between plays and movies, and Passage 2 does not mention any “other art forms” at all. ● (D) is wrong because only Passage 2 discusses “how one’s perception of the theater may develop over time”—this subject is unmentioned in Passage 1. ● (E) is wrong because there is no reference in either passage to “the experience of reading a play.” Correct answer: (A) / Difficulty level: Easy Some questions assess your comprehension of information that is directly stated in a passage. 11. The “happenings” mentioned in line 14 refer to the (A) work undertaken to produce a movie (B)events occurring in the street outside the theater (C) fantasies imagined by a child (D) activity captured on the movie screen (E) story unfolding on the stage To answer this question correctly, you have to understand lines 11–15, a rather complex sentence that makes an important distinction in Passage 1. The author indicates that, unlike plays, movies leave “the mind’s grasp of reality intact,” because the “happenings” in a movie are not occur- ring in the actual theater. Instead, images are projected on a screen in the theater. Thus (D) is the correct answer; the word “happenings” refers to the “activity captured on the movie screen.” ● (A) and (B) are wrong because, when you insert them in place of the word “happenings,” the sen- tence in lines 11-15 makes no sense. ● (C) is wrong; even if the movies being referred to include “fantasies” in them, they are not “imagined by a child” but are actually projected on the movie screen. ● (E) is wrong because, in line 14, “happenings” refers to the “story unfolding” in a mo vie,not “on the stage.” Correct answer: (D) / Difficulty level: Medium You may be asked to recognize the author’s tone or attitude in a particular part of a passage, or in the passage as a whole. 12. In the final sentence of Passage 2 (“I thought ...in me”), the author expresses (A) exultation (B) vindication (C) pleasure (D) regret (E) guilt Even though this question focuses on a single sentence, you must understand the context in which the statement occurs in order to determine the feeling expressed by the author. In the second paragraph of Passage 2, the author states that the experience of attending a play at age 12 or 13 was much different than at age 6. “The same things were there materially” in the theater, but the older child knew SAT Preparation Booklet 12 85 90 95

SAT Preparation Booklet 13 much more than the younger one about what was going on. Ironically, this increased knowledge actually decreased the author’s pleasure in attending the play. “In that interval what had I not lost,” the author exclaims in line 78. Where the younger child saw nobles in “the court of ancient Persia,” the older child saw “men and women painted.” Thus the final sentence of Passage 2 expresses “regret” con- cerning the changes that “those many centuries—those six short years—had wrought” in the author. (D) is the correct answer. ● (A) and (C) are incorrect because the author does not feel “exultation” about or take “pleasure” in the “alteration” that has occurred; on the contrary, the author laments it. ● (B) is incorrect because there is no expression of “vindication” in the final sentence; the author is not trying to justify, support, or defend the experi- ences described in the passage but rather explain the changes that have occurred due to the passage of time. ● (E) is incorrect because, even though the final sen- tence states that the “fault” was not in the actors but in the now more knowledgeable child, the author feels no “guilt” about the change. There is no way to avoid the passage of time (and the learn- ing that goes along with it). Aging is not the child’s “fault,” but the loss of a youthful sense of wonder and innocence can still cause regret. Correct answer: (D) / Difficulty level: Hard Some questions require you to determine and compare the primary purpose or main idea expressed in each passage. 13. Which of the following best describes the difference between Passages 1 and 2 ? (A) Passage 1 remembers an event with fondness, while Passage 2 recalls a similar event with bitter detachment. (B) Passage 1 considers why the author responded to the visit as he did, while Passage 2 supplies the author’s reactions without further analysis. (C) Passage 1 relates a story from a number of different perspectives, while Passage 2 maintains a single point of view. (D) Passage 1 treats the visit to the theater as a disturbing episode in the author’s life, while Passage 2 describes the author’s visit as joyful. (E) Passage 1 recounts a childhood experience, while Passage 2 examines how a similar experience changed over time.This question asks you to do two things: first, understand the overall subject or purpose of each passage; second, rec- ognize an important “difference between” the two. The cor- rect answer is (E) because the entire first passage does indeed tell the story of a particular “childhood experi- ence”—a trip to the theater—while the second passage describes two different trips to the theater and how the “experience changed over time.” ● (A) is wrong because there is neither bitterness nor “detachment” in Passage 2. In fact, the first paragraph of Passage 2 expresses excitement and “enchantment,” and the second paragraph expresses disappointment and regret. ● (B) is wrong because Passage 2 includes a great deal more than just “the author’s reactions” to visiting the theater; most of the second paragraph provides “further analysis” of what had changed and why the reactions to the two visits were so different. ● (C) is wrong because it r everses the two narrative approaches in this pair of passages. Passage 1 “maintains a single point of view,” that of the youthful first-time theatergoer, whereas the author of Passage 2 presents at least two “different per- spectives,” that of the enchanted six year old and of the older child returning to the theater. ● (D) is wrong because the author of Passage 1 does not find his first visit to the theater “disturbing” in a negative way. Although he feels “shock” when the curtain goes up and anxiety during the play, these responses merely indicate how effective and “real” the performance was for him. In the end, the child and his mother walked “happily” out of the theater. Correct answer: (E) / Difficulty level: Easy

SAT Preparation Booklet 14 The Math Section The math section of the SAT contains two types of questions: ● standard multiple-choice (44 questions) ● student-produced response questions that provide no answer choices (10 questions) Some questions are like the questions in math textbooks. Others ask for original thinking and may not be as familiar to you. Calculator Policy We recommend that you bring a calculator to use on the math section of the SAT. Every question on the test can be solved without a calculator; however, using a calculator on some questions may be helpful to you. A scientific or graphing calculator is recommended. Acceptable Calculators Calculators permitted during testing are: ● graphing calculators ● scientific calculators ● four–function calculators (not recommended) Ifyou have a calculator with characters that are 1 inch or higher, or if your calculator has a raised display that might be visible to other test-takers, you will be seated at the dis- cretion of the test supervisor. You will not be allowed to share calculators. You will be dismissed and your scores canceled if you use your calcula- tor to share information during the test or to remove test questions or answers from the test room. Unacceptable Calculators Unacceptable calculators are those that: ● use QWERTY (typewriter-like) keypads ● require an electrical outlet ● “talk” or make unusual noises ● use paper tape ● are electronic writing pads, pen input/stylus-driven devices, pocket organizers, cell phones, power- books, or handheld or laptop computers Approaches to the Math Section ● Familiarize yourself with the directions ahead of time. Also learn how to complete the grids for student-produced response questions. ● Ask yourself the following questions before you solve each problem: What is the question asking? What do I know? ● Limit your time on any one question. All questions are worth the same number of points. If you need a lot of time to answer a question, go on to the next one. Later, you may have time to return to the question you skipped. ● Keep in mind that questions are generally arranged from easy to hard. Within any group of ques- tions—for example, the multiple-choice questions —the easier ones come first and the questions become more difficult as you move along. ● Don't make mistakes because of carelessness. No matter how frustrated you are, don't pass over questions without at least reading them, and be sure to consider all the choices in each question. If you're careless, you could choose the wrong answers even on easy questions. ● Work out the problems in your test booklet. You will not receive credit for anything written in the booklet, but you will be able to check your work easily later. ● Eliminate choices. If you don't know the correct answer to a question, try eliminating wrong choic- es. It's sometimes easier to find the wrong answers than the correct one. On some questions, you can eliminate all the incorrect choices. Draw a line through each choice as you eliminate it until you have only the one correct answer left. ● Keep in mind that on student-produced response (grid-in) questions you don’t lose points for wrong answers. Make an educated guess if you don't know the answer. ● For student-produced response questions, always enter your answer on the grid. Remember: for grid- in questions only answers entered on the grid are scored. Your handwritten answer at the top of the grid isn't scored. However, writing your answer at the top of the grid may help you avoid gridding errors. Important: For grid-in questions only answers entered on the grid are scored. Your hand- written answer at the top of the grid is not scored.

SAT Preparation Booklet 15 MATH REVIEW MATHEMATICS CONTENT For the new SAT, the mathematics content level of the test will be raised to include more advanced topics. The follow- ing math concepts will be covered beginning with the March 2005 test. Number and Operation ● Arithmetic word problems (including percent, ratio, and proportion) ● Properties of integers (even, odd, prime numbers, divisibility, etc.) ● Rational numbers ● Logical reasoning ● Sets (union, intersection, elements) ● Counting techniques ● Sequences and series (including exponential growth) ● Elementary number theory Algebra and Functions ● Substitution and simplifying algebraic expressions ● Properties of exponents ● Algebraic word problems ● Solutions of linear equations and inequalities ● Systems of equations and inequalities ● Quadratic equations ● Rational and radical equations ● Equations of lines ● Absolute value ● Direct and inverse variation ● Concepts of algebraic functions ● Newly defined symbols based on commonly used operations Geometry and Measurement ● Area and perimeter of a polygon ● Area and circumference of a circle ● Volume of a box, cube, and cylinder ● Pythagorean Theorem and special properties of isosceles, equilateral, and right triangles ● Properties of parallel and perpendicular lines ● Coordinate geometry ● Geometric visualization ● Slope ● Similarity ● Transformations Data Analysis, Statistics, and Probability ● Data interpretation ● Statistics (mean, median, and mode) ● Probability ARITHMETIC AND ALGEBRAIC CONCEPTS ● Integers:...,-4,-3,-2,-1,0,1,2,3,4,... (Note: zero is neither positive nor negative.) ● Consecutive Integers:Integers that follow in sequence; for example, 22, 23, 24, 25. Consecutive integers can be more generally represented by n, ,,,... ● Odd Integers:...,-7,-5,-3,-1,1,3,5,7,..., ,...where is an integer ● Even Integers:...,-6,-4,-2,0,2,4,6,..., , ...,where is an integer (Note: zero is an even integer.) ● Prime Numbers:2,3,5,7,11,13,17,19,... (Note: 1 is not a prime and 2 is the only even prime.) ● Digits:0, 1, 2, 3, 4, 5, 6, 7, 8, 9 (Note: the units digit and the ones digit refer to the same digit in a number. For example, in the number 125, the 5 is called the units digit or the ones digit.) Percent Percent means hundredths, or number out of 100. For example, 40 percent means or 0.40 or . Problem 1:Ifthe sales tax on a $30.00 item is $1.80, what is the sales tax rate? Solution: is the sales tax rate. Percent Increase / Decrease Problem 2:Ifthe price of a computer was decreased from $1,000 to $750, by what percent was the price decreased? Solution:The price decrease is $250.The percent decrease is the value ofnin the equation = . The value ofnis 25, so the price was decreased by 25%. Note: = ; = . n 100 decrease original n 100 increase original n 100 250 1000 , n=66,%so $. $ .180 10030 00 =×n 2 5 40 1 00 k 2k k 21k+ n+3 n+2 n+1

SAT Preparation Booklet 16 MATH REVIEW Average An averageis a statistic that is used to summarize data. The most common type of average is the arithmetic mean. The average (arithmetic mean) of a list ofnnumbers is equal to the sum of the numbers divided by n. For example, the mean of 2, 3, 5, 7, and 13 is equal to When the average of a list ofnnumbers is given, the sum of the numbers can be found. For example, if the average of six numbers is 12, the sum of these six numbers is The medianof a list of numbers is the number in the mid- dle when the numbers are ordered from greatest to least or from least to greatest. For example, the median of 3, 8, 2, 6, and 9 is 6 because when the numbers are ordered, 2, 3, 6, 8, 9, the number in the middle is 6. When there is an even number of values, the median is the same as the mean of the two middle numbers. For example, the median of 6, 8, 9, 13, 14, and 16 is the mean of 9 and 13, which is 11. The modeof a list of numbers is the number that occurs most often in the list. For example, 7 is the mode of 2, 7, 5, 8, 7, and 12. The numbers 2, 4, 2, 8, 2, 4, 7, 4, 9, and 11 have two modes, 2 and 4. Note:On the SAT, the use of the word average refers to the arithmetic mean and is indicated by “average (arithmetic mean).” The exception is when a question involves average speed (see problem below). Questions involving median and mode will have those terms stated as part of the ques- tion’s text. Average Speed Problem:José traveled for 2 hours at a rate of 70 kilometers per hour and for 5 hours at a rate of 60 kilome- ters per hour. What was his average speed for the 7-hour period? Solution:In this situation, the average speed is The total distance is 2 hr + 5 hr = 440 km. The total time is 7 hours. Thus, the average speed was = kilometers per hour. 626 7 440 7km hr 60km hr      70km hr      total distance total time 12 6 72×or . 235713 56 +++ + = Note:In this example, the average speed is not the average of the two separate speeds, which would be 65 kilometers per hour. Factoring You may need to apply these types of factoring: Probability Probability refers to the chance that a specific outcome can occur. It can be found by using the following definition when outcomes are equally likely: For example, if a jar contains 13 red marbles and 7 green marbles, the probability that a marble selected from the jar at random will be green is Ifa particular outcome can never occur, its probability is 0. Ifan outcome is certain to occur, its probability is 1. In general, ifpis the probability that a specific outcome will occur, values ofpfall in the range . Probability may be expressed as either a decimal or a fraction. Functions A function is a relation in which each element of the domain is paired with exactlyone element of the range. On the SAT, unless otherwise specified, the domain of any function is assumed to be the set of all real numbers for which is a real number. For example, if ,the domain of is all real numbers greater than or equal to . For this function, 14 is paired with 4, since . Note:the symbol represents the positive, or principal, square root. For example, , not ± 4. Exponents You should be familiar with the following rules for exponents on the SAT. 16 4= f14 14 2 16 4() =+= = −2 f fx x() =+2 fx() x f 01≤≤p 7 7137 20035 +=or . Number of ways that a specific outcome can occur Total number of possible outcomes 25321 3 2xx x x+−= − () + ( ) xx x x x22 21 1 1 1 ++=+ () + () =+() xxx2 111 −= + () − () xxxx2 22 += + ()

SAT Preparation Booklet 17 MATH REVIEW For all values of : For all values of For example, Note:For any nonzero number Sequences Two common types of sequences that appear on the SAT are arithmetic and geometric sequence. An arithmetic sequenceis a sequence in which successive terms differ by the same constant amount. For example: 3, 5, 7, 9,...is an arithmetic sequence. A geometric sequenceis a sequence in which the ratio of successive terms is a constant. For example: 2, 4, 8, 16,...is a geometric sequence. A sequence may also be defined using previously defined terms. For example, the first term of a sequence is 2, and each successive term is 1 less than twice the preceding term.This sequence would be 2, 3, 5, 9, 17,... On the SAT, explicit rules are given for each sequence. For example, in the geometric sequence above, you would not be expected to know that the 5th term is 32 unless you were given the fact that each term is twice the preceding term.For sequences on the SAT, the first term is never referred to as the zeroth term. Variation Direct Variation:The variable is directly proportional to the variable if there exists a nonzero constant such that . Inverse Variation:The variable is inversely proportional to the variable if there exists a nonzero constant such that xk yxy k ==or . k y x ykx= k x y xx, 0 1 = xx 2 32 3 = xx a ba b = x x a a −=1 x yx y a a a     = x xxa bab = − ab x y,, ,≠≠00: xy x y a a a () = ⋅ xxab ab() = ⋅ x xx ab ab⋅ = + abx y,,, Absolute Value The absolute value of is defined as the distance from to zero on the number line. The absolute value of is written in the form . For all real numbers : For example: GEOMETRIC CONCEPTS Figures that accompany problems are intended to provide information useful in solving the problems. They are drawn as accurately as possible EXCEPT when it is stated in a particular problem that the figure is not drawn to scale. In general, even when figures are not drawn to scale, the relative positions of points and angles may be assumed to be in the order shown. Also, line segments that extend through points and appear to lie on the same line may be assumed to be on the same line. The text “N ote:Figure not drawn to scale” is included with the figure when degree measures may not be accurately shown and specific lengths may not be drawn proportionally. The following examples illustrate what information can and cannot be assumed from figures. Example 1: Since and are line segments, angles and are vertical angles. Therefore, you can conclude that .Even though the figure is drawn to scale, you should NOT make any other assumptions without addi- tional information. For example, you should NOT assume that or that the angle at vertex is a right angle even though they might look that way in the figure. E AC CD= xy= DC E AC B BE AD 22 20 22 20 00=> −= −< =       , ,sin ce sin ce xxx xx =≥ −<   , ,if if0 0 x x x x x

SAT Preparation Booklet 18 MATH REVIEW Example 2: N ote:Figure not drawn to scale A question may refer to a triangle such as above. Although the note indicates that the figure is not drawn to scale, you may assume the following: ● and are triangles. ● is between and . ● ,,and are points on a line. ● The length of is less than the length of . ● The measure of angle is less than the meas- ure of angle . You may notassume the following: ● The length of is less than the length of . ● The measures of angles and are equal. ● The measure of angle is greater than the measure of angle . ● Angle is a right angle. Geometric Skills and Concepts Properties of Parallel Lines 1. If two parallel lines are cut by a third line, the alternate interior angles are congruent. In the figure above, cx wd==and  m k a°b° c°d° w°x° y°z° ABC DB C ABD BDA BA D DC AD ABC ABD AC AD C D A C A D DBC ABD ABC 2. If two parallel lines are cut by a third line, the cor- responding angles are congruent. In the figure, 3. If two parallel lines are cut by a third line, the sum of the measures of the interior angles on the same side of the transversal is 180°. In the figure, Angle Relationships 1. The sum of the measures of the interior angles of a triangle is 180°. In the figure above, 2. When two lines intersect, vertical angles are congruent. In the figure, 3. A straight angle measures 180°. In the figure, 4. The sum of the measures of the interior angles of a polygon can be found by drawing all diagonals of the polygon from one vertex and multiplying the number of triangles formed by 180°. Since the polygon is divided into 3 triangles, the sum of the measures of the angles is 3 180°, or 540°. Unless otherwise noted in the SAT, the term “polygon” will be used to mean a convex polygon, that is, a polygon in which each interior angle has a measure of less than 180°. A polygon is “regular” if all sides are congruent and all angles are congruent. × zz=+=130 50 180because y=50 xx=++=70 60 50 180because 60°50° x° y° z° cw d x+= +=180 180and awcybx d z=== =,, and

SAT Preparation Booklet 19 MATH REVIEW Side Relationships 1. Pythagorean Theorem: In any right triangle ,where cis the length of the longest side and aand bare the lengths of the two shorter sides. To find the value of ,use the Pythagorean Theorem. 2. In any equilateral triangle, all sides are congruent and all angles are congruent. Because the measure of the unmarked angle is 60°, the measures of all angles of the triangle are equal; and, therefore, the lengths of all sides of the triangle are equal: 3. In an isosceles triangle, the angles opposite con- gruent sides are congruent. Also, the sides opposite congruent angles are congruent. In the figures below, . 4. In any triangle, the longest side is opposite the largest angle, and the shortest side is opposite the smallest angle. In the figure below, . 5. Two polygons are similarif and only if the lengths of their corresponding sides are in the same ratio and the measures of their corresponding angles are equal. abc

SAT Preparation Booklet 20 MATH REVIEW Coordinate Geometry 1. In questions that involve the and to the right of the are positive and to the left of the are negative. Similarly, above the are positive and below the are negative. In an ordered pair , the is written first. For example, in the pair , the is and the is 3. 2. Slope of a line A line that slopes upward as you go from left to right has a positiveslope. A line that slopes down- ward as you go from left to right has a negative slope. A horizontal line has a slope of zero. The slope of a vertical line is undefined. Parallel lines have the same slope. The product of the slopes of two perpendicular lines is , provided the slope of each of the lines is defined. For example, any line perpendicular to line above has a slope of . 4 3  −1 Slope of=−− −−=− 12 223 4 () Slope ofPQ==4 2 2 ==rise runchange in - coordinates chan geiny x - -coordinates y-coor dinate −2 x-coordinate −()23, x-coor dinate xy,() x-axis y-va lues x-axis y-values y-axis x-va lues y-axis x-va lues y-axes, x- 1 1 x (–2, 3) y O The equation of a line can be expressed as ,where is the slope and is the intercept. Since the slope of line is , the equation of line can be expressed as . Since the point is on the line, must satisfy the equa- tion. Hence, and the equa- tion of line is . 3. The equation of a parabola can be expressed as where the vertex of the parabola is at the point and . If ,the parabola opens upward; and if , the parabola opens downward. The parabola above has its vertex at . Therefore, and . The equation can be represented by . Since the parabola opens downward, we know that . To find the value ofa,you also need to know another point on the parabola. Since we know the parabola passes through the point must satisfy the equation. Hence, .Therefore, the equation for the parabola is . yx=− + () + 1 324 2 11241 3 2 =+ () +=− aa,so xy==11and 11,,() a

SAT Preparation Booklet 21 Multiple-Choice Questions The questions that follow will give you an idea of the type of mathematical thinking required to solve problems on the SAT. First, try to answer each question yourself and then read the solutions that follow. These solutions may give you new insights into solving the problems or point Directions Sample Questions Below are seven examples of standard multiple-choice questions. Following each question, you will find one or two solutions. 1. A special lottery is to be held to select the student who will live in the only deluxe room in a dormi- tory.There are 100 seniors, 150 juniors, and 200 sophomores who applied. Each senior’s name is placed in the lottery 3 times; each junior’s name, 2 times; and each sophomore’s name, 1 time. What is the probability that a senior’s name will be chosen? (A) (B) (C) (D) (E) Correct answer: (D) / Difficulty level: Medium 1 2 3 8 2 7 2 9 1 8 To determine the probability that a senior’s name will be chosen, you must determine the total number of seniors’ names that are in the lottery and divide this number by the total number of names in the lottery. Since each senior’s name is placed in the lottery 3 times, there are seniors’ names. Likewise, there are juniors’ names and sopho- mores’ names in the lottery. The probability that a senior’s name will be chosen is 300 300 300 200300 8003 8 ++==. 1200 200 ×= 21 50300 ×= 3100 300 ×= Notes 1. The use of a calculator is permitted. 2. All numbers used are real numbers. 3. Figures that accompany problems in this test are intended to provide information useful in solving the problems. They are drawn as accurately as possible EXCEPT when it is stated in a specific problem that the figure is not drawn to scale. All figures lie in a plane unless otherwise indicated. 4. Unless otherwise specified, the domain of any function f is assumed to be the set of all real numbers x for which f(x) is a real number. h r r h b A = r 2 C = 2 r A = bh V = wh V = r 2h The number of degrees of arc in a circle is 360. The sum of the measures in degrees of the angles of a triangle is 180. b ac c2 = a 2 + b 2 Special Right Triangles x 3 2x x 60° 30° s s45° 45° A = w w wh Reference Information 2 s 1 2 out techniques you’ll be able to use again. Most problems can be solved in a variety of ways, so don’t be concerned if your method is different from the one given. Note that the directions indicate that you are to select the best of the choices given. For this section, solve each problem and decide which is the best of the choices given. Fill in the corresponding circle on the answer sheet. You may use any available space for scratchwork.

SAT Preparation Booklet 22 NOONTIME TEMPERATURES IN HILO, HAWAII Mon Tue Wed Thu Fri Sat Sun 66 78 75 69 78 77 70 2. The table above shows the temperatures, in degrees Fahrenheit, in a city in Hawaii over a one-week period. Ifmrepresents the median temperature,f represents the temperature that occurs most often, and arepresents the average (arithmetic mean) of the seven temperatures, which of the following is the correct order ofm,f,and a ? (A) (B) (C) (D) (E) Correct answer: (A) / Difficulty level: Medium To determine the correct order ofm,f,and a,it is helpful to first place the seven temperatures in ascending order as shown below. 66 69 70 75 77 78 78 The median temperature is the middle temperature in the ordered list, which is 75, so m= 75. The temperature that occurs most often, or the mode, is 78, so f= 78. To deter- mine the average, you can add the seven numbers together and divide by 7. However, you can determine the relation- ship between the average and the median by inspection. The three numbers greater than 75 are closer to 75 than are the three numbers smaller than 75. Therefore, the average of the seven numbers will be less than 75. The correct order ofm,f,and ais . 3. The projected sales volume of a video game cartridge is given by the function , where sis the number of cartridges sold, in thou- sands;pis the price per cartridge, in dollars; and a is a constant. If according to the projections, 100,000 cartridges are sold at $10 per cartridge, how many cartridges will be sold at $20 per cartridge? (A) 20,000 (B) 50,000 (C) 60,000 (D) 150,000 (E) 200,000 Correct answer: (C) / Difficulty level: Medium sp pa() = + 3000 2 am f

SAT Preparation Booklet 23 5. If two sides of the triangle above have lengths 5 and 6, the perimeter of the triangle could be which of the following? I. 11 II. 15 III. 24 (A) I only (B) II only (C) III only (D) II and III only (E) I, II, and III Correct answer: (B) / Difficulty level: Medium In questions of this type, statements I, II, and III should each be considered ind epend entl yof the others. You must determine which of those statements couldbe true. ● Statement I cannot be true. The perimeter of the triangle cannot be 11 since the sum of the two given sides is 11 without even considering the third side of the triangle. ● Continuing to work the problem, you see that in II, if the perimeter were 15, then the third side of the triangle would be 15 - (6 + 5), or 4. A triangle can have side lengths of 4, 5, and 6. So the perimeter of the triangle could be 15. ● Finally, consider whether it is possible for the trian- gle to have a perimeter of 24. In this case, the third side of the triangle would be 24 - (6 + 5) = 13. The third side of this triangle cannot be 13, since the sum of the other two sides is not greater than 13. By the Triangle Inequality, the sum of the lengths of any two sides of a triangle must be greater than the length of the third side. So the correct answer is II only. Note: Figure not drawn to scale. ab c 6. If , what is the value ofm ? (A) (B) (C) (D) (E) Correct answer: (C) / Difficulty level: Medium Since can be written as and can be written as , the left side of the equation is ,the value ofmis . 7. Ifkis divisible by 2, 3, and 15, which of the follow- ing is also divisible by these numbers? (A) (B) (C) (D) (E) Correct answer: (D) / Difficulty level: Medium Since kis divisible by 2, 3, and 15,kmust be a multiple of 30, as 30 is the least common multiple of 2, 3, and 15. Some multiples of 30 are 0, 30, 60, 90, and 120. ● Ifyou add two multiples of 30, the sum will also be a multiple of 30. For example, 60 and 90 are multi- ples of 30 and their sum, 150, is also a multiple of 30. ● Ifyou add a multiple of 30 to a number that is not a multiple of 30, the sum will not be a multiple of 30. For example, 60 is a multiple of 30 and 45 is not. Their sum, 105, is not a multiple of 30. ● The question asks which answer choice is divisible by 2, 3, and 15; that is, which answer choice is a multiple of 30. All the answer choices are in the form of “kplus a number.” Only choice (D), ,is the sum ofkand a multiple of 30. The sum ofkand 30 is also a multiple of 30, so the correct answer is choice (D). k+30 k+45 k+30 k+20 k+15 k+5 −5 2 xx x x x 1 21 25 25 2 33 ⋅= = −−             −− .Since= =x m x−3 1 3x x 1 2 x −3 2 −2 −5 2 −3 −7 2 xx xx m >=1 3 and

SAT Preparation Booklet 24 Student-Produced Response Questions Questions of this type have no answer choices provided. Instead, you must solve the problem and fill in your answer on a special grid. Ten questions on the test will be of this type. It is very important for you to understand the directions for entering answers on the grid!You will lose valuable testing time if you read the directions for the first time when you take the test. The directions are fairly simple, and the grid- ding technique is similar to the way other machine- readable information is entered on forms.A primary advantage of this format is that it allows you to enter the form of the answer that you obtain, whether whole number, decimal, or fraction. For example, if you obtain 2/5, you can grid 2/5. If you obtain .4, you can grid .4. Generally, you should grid the form of the answer that you obtain naturally in solving the problem. The grid will only hold numbers that range from 0 to 9999. Decimals and fractions can also be gridded. Below are the actual directions that you will find on the test—read them carefully. Each of the remaining questions requires you to solve the problem and enter your answer by marking the circles in the special grid, as shown in the examples below. You may use any available space for scratchwork. Decimal Answers: If you obtain a decimal answer with more digits than the grid can accommodate, it may be either rounded or truncated, but it must fill the entire grid. For example, if you obtain an answer such as 0.6666..., you should record your result as .666 or .667. A less accurate value such as .66 or .67 will be scored as incorrect. Acceptable ways to grid are: 2 3 Note: You may start your answers in any column, space permitting. Columns not needed should be left blank. Mark no more than one circle in any column. Because the answer sheet will be machine- scored, you will receive credit only if the circles are filled in correctly. Although not required, it is suggested that you write your answer in the boxes at the top of the columns to help you fill in the circles accurately. Some problems may have more than one correct answer. In such cases, grid only one answer. No question has a negative answer. Mixed numbers such as 3 must be gridded as 3.5 or 7 2. (If is gridded, it will be interpreted as , not 3 .) 1 2 1 2 31 2 • • • • • • 1 2 3 4 5 60 1 2 3 4 5 60 1 2 3 4 5 60 1 2 3 4 5 61 2 3 4 5 6¥ 0 1 2 3 4 50 1 2 3 4 50 1 2 3 4 51 2 3 4 5 6¥ 0 1 2 3 4 50 1 2 3 4 50 1 2 3 4 5 6 Answer: 2.5 Fraction line Decimal point Write answer in boxes. Grid in result. Answer: 201 Either position is correct. 7 12 1 2 3 4 5 6 7 8 90 1 2 3 4 5 6 7 8 90 1 2 3 4 5 6 7 8 90 1 2 3 4 5 6 7 8 91 2 3 4 5 6 7 8 90 1 2 3 4 5 6 7 8 90 1 2 3 4 5 6 7 8 90 1 2 3 4 5 6 7 8 91 2 3 40 1 2 3 40 1 2 3 40 1 2 3 40 1 2 3 40 1 2 30 1 2 3 1 2 3 Answer: •

SAT Preparation Booklet 25 9. For all positive integers aand b,let a bbe defined by a bWhat is the value of 42? The words “let a bbe defined by ” tell you that the symbol  is not supposed to represent a common mathematical operation but one that is made up for this question. To evaluate 42,you would substitute 4 for aand 2 for bin the expression This gives , which equals .The answer may be entered in the grid as 17/3 or as 5.66 or 5.67. Difficulty level: Medium 10. Of the 6 courses offered by the music department at her college, Kay must choose exactly 2 of them. How many different combinations of 2 courses are possible for Kay if there are no restrictions on which 2 courses she can choose? There are 6 courses offered; let us refer to them as 1, 2, 3, 4, 5, and 6. One way to find the number of combinations is to list all possible pairings. They are 1-2, 1-3, 1-4, 1-5, 1-6, 2-3, 2-4, 2-5, 2-6, 3-4, 3-5, 3-6, 4-5, 4-6, and 5-6. There are 15 combinations. Note that 1-2 and 2-1 represent the same combination. 1 2 3 4 5 6 7 80 1 2 3 4 5 6 7 80 1 2 3 4 5 6 7 80 1 2 3 4 5 6 7 8 9999 5 1 17 3 41 412+ − a ab+ −1 1. =+ − a a b 1 1. Sample Questions Below are five examples of student-produced response questions. Following each question, you will find a solution and several ways to enter the correct answer. 8. What value ofx satisfies both of the equations above? Since , the value of or The two values ofxthat satisfy the first equation are 3 and . or The two values ofxthat satisfy the second equation are and . You are asked to find the value ofxthat satisfies b oth equations. That value is . The answer can be entered in the grid as 1/2 or .5. Difficulty level: Hard 1 2 1 2 1 4 38 1 84 1 2 −=− = = x x x 38 1 82 1 4 −= = = x x x Sin ce the value ofiseither 38 1 38 1−= −xx,or−1. 1 2 47 5 42 1 2 x x x −=− = = 475 412 3 x x x −= = = 47 5 5x−−is either or . 475x−= 1 2 3 4 5 6 7 8 90 1 2 3 4 5 6 7 8 90 1 2 3 4 5 6 7 8 90 1 2 3 4 5 6 7 8 9 5 1 2 3 4 5 6 7 8 90 1 2 3 4 5 6 7 8 90 1 2 3 4 5 6 7 8 90 1 2 3 4 5 6 7 8 9 12 / 475 38 1x x −= −= 1 2 3 4 5 6 7 8 90 1 2 3 4 5 6 7 8 90 1 2 3 4 5 6 7 8 90 1 2 3 4 5 6 7 8 9 7 13 / 1 2 3 4 5 6 7 8 90 1 2 3 4 5 6 7 8 90 1 2 3 4 5 6 7 8 90 1 2 3 4 5 6 7 8 9 6 56 1 2 3 4 5 6 7 8 90 1 2 3 4 5 6 7 8 90 1 2 3 4 5 6 7 8 90 1 2 3 4 5 6 7 8 9 7 56

SAT Preparation Booklet 26 You could also notice that there are 5 pairings that start with course 1 and 4 additional pairings that start with course 2, and so forth. The total number of combinations is You could also solve the problem by noting that the total number of permutations (that is, the number of different ways 2 of 6 courses could be selected) is 6 for the first course selected times 5 for the second course selected, or To find the number of combinations, you must divide the number of permutations by the number of arrangements. For each pair of courses A-Bselected, the arrangement B-Ais also possible. Therefore, there are 2 arrangements. So, the number of combinations is Difficulty level: Medium 11. Since , substituting for xinto the function yields or or . Since or Therefore . Another way to solve the question would be to use a dummy variable k.For example, let . Since and So , and therefore, Sin ceortk t t=− = =14 1,. kk==52or . kk−() − () = 520 ft fk+() = () = 10 0,. kt=+1 fk k k k k() =−+=− () − () 2 710 5 2. kt=+1 tt==14or tt−() −() = 140. ft t t+() =−+= 10 540 2,, ft t t+() =−+ 154 2 ft t t t+() =++() −+() + 1217710 2 , ft t t+() =+() −+ () + 117110 2 , t+() 1 fx x x() =−+ 2 710 1 2 3 4 5 6 7 8 90 1 2 3 4 5 6 7 8 90 1 2 3 4 5 6 7 8 90 1 2 3 4 5 6 7 8 9 4 1 2 3 4 5 6 7 8 90 1 2 3 4 5 6 7 8 90 1 2 3 4 5 6 7 8 90 1 2 3 4 5 6 7 8 9 1 If and wh a t is o nfx x x ft () =−+ + () = 2 710 10, e epossible value oft? 30 2 15 ÷ =. 65 30×=. 5432115++++=. This question asks for one possiblevalue ofx.Either 1 or 4 (not both) satisfy the question being asked. Choose only one correct answer to enter in the grid. When there is a range of possible correct answers, your gridded responsemust lie within the range. For example, consider a problem for which all numbers between 4 and 5, exclusive, are correct answers. For this problem, although 4.0002 is within the range , its rounded value 4.00 is not within the range and would not therefore be considered a correct answer to the problem. Difficulty level: Hard 12. Three parallel lines in a plane are intersected by a fourth line, forming twelve angles. If one of the angles has measure 28˚, how many of the other eleven angles have measure 28˚ ? Drawing the figure described in the problem will help you visualize the correct solution to the problem. The figure below shows three parallel lines intersected by a fourth line. The acute angle is labeled 28˚. Using the fact that vertical angles and alternate interior angles are equal, you can put a check showing the othe r angles in the figure that also measure 28˚, as shown below. There are 5 other angles that measure 28˚. Therefore, the correct answer to this problem is 5. The number 5 can be gridded in any of the four columns on the answer grid. Difficulty level: Easy 28° 28° 1 2 3 4 5 6 7 8 90 1 2 3 4 5 6 7 8 90 1 2 3 4 5 6 7 8 90 1 2 3 4 5 6 7 8 9 5 45

SAT Preparation Booklet 27 The Writing Section The new writing section includes both multiple-choice questions and a direct writing measure in the form of an essay. The multiple-choice questions will test: ● improving sentences (25 questions) ● identifying sentence errors (18 questions) ● improving paragraphs (6 questions) Multiple-choice writing questions are more about the mechanics of writing than the process of composing. They will assess your ability to: ● use language that is consistent in tenses and pronouns ● understand parallelism, noun agreement, and subject-verb agreement ● understand how to express ideas logically ● avoid ambiguous and vague pronouns, wordiness, improper modification, and sentence fragments ● understand proper coordination and subordina- tion, logical comparison, and modification and word order You will not be asked to define or use grammatical terms, and spelling and capitalization will not be tested. Note:Calculators may not be on your desk or be used during the writing section of the SAT. Approaches to the Multiple- Choice Writing Questions ● Read the directions carefully and then follow them. ● Eliminate the choices you are sure are wrong when you are not sure of the answer. Make an educated guess from the remaining choices. Improving Sentences This question type measures your ability to: ● recognize and correct faults in usage and sentence structure ● recognize effective sentences that follow the con- ventions of standard written English Directions The following sentences test correctness and effective- ness of expression. Part of each sentence or the entire sentence is underlined; beneath each sentence are five ways of phrasing the underlined material. Choice A repeats the original phrasing; the other four choices are different. If you think the original phrasing pro- duces a better sentence than any of the alternatives, select choice A; if not, select one of the other choices. In making your selection, follow the requirements of standard written English; that is, pay attention to grammar, choice of words, sentence construction, and punctuation. Your selection should result in the most effective sentence—clear and precise, without awkward- ness or ambiguity. EXAMPLE: Laura Ingalls Wilder published her first book and she was sixt y-fi ve y ear s old the n. (A) and she was sixty-five years old then (B) when she was sixty-five (C) at age sixty-five years old (D) upon the reaching of sixty-five years (E) at the time when she was sixty-five Answering Improving Sentences Questions Look carefully at the underlined portion of the sentence because it may have to be revised. Keep in mind that the rest of the sentence stays the same. Follow the two outlined steps below in answering each Improving Sentences question. Step 1:Read the entire sentence carefully but quickly and ask yourself whether the underlined portion is correct or whether it needs to be revised. In the example above, connecting the two ideas (“Laura Ingalls Wilder published her first book”)and (“she was sixty-five years old then”)with the word “and”indicates that the two ideas are equally important. The word “and” should be replaced to establish the relationship between the two ideas. Step 2:Read choices (A) through (E), replacing the under- lined part with each answer choice to determine which revision results in a sentence that is clear and precise and meets the requirements of standard written English. Remember that choice (A) is the same as the underlined portion. Even if you think that the sentence does not require correction and choice (A) is the correct answer, it is a good idea to read each choice quickly to make sure. ● The word “and”indicates that the two ideas it con- nects are equally important. No. ● Replacing the word “and” with “when”clearly expresses the information that the sentence is intended to convey by relating Laura Ingalls Wilder’s age to her achievement. Yes, but continue to look at the other revisions. ● Using the word “at”results in a phrase that is not idiomatic. No. ● The phrase “upon the reaching of ”also results in a phrase that is not idiomatic. No. ● The phrase “at the time when she was sixty-five years old”is awkward and wordy. No. Correct answer: (B) / Difficulty level: Easy A BC D E

SAT Preparation Booklet 28 Sample Questions 1. Sc enes fr om the e veryday li ves o fAfrican Ame ricans, whic h ar e r ealist ical ly d epic ted in the paint ing s o f H enry Ossa wa T anne r. (A) Scenes from the everyday lives of African Americans, which are realistically depicted in the paintings of Henry Ossawa Tanner. (B) Scenes from the everyday lives of African Americans being realistically depicted in the paintings of Henry Ossawa Tanner. (C) The paintings of Henry Ossawa Tanner realistically depict scenes from the everyday lives of African Americans. (D) Henry Ossawa Tanner, in his realistic paintings, depicting scenes from the everyday lives of African Americans. (E) Henry Ossawa Tanner, whose paintings realistically depict scenes from the everyday lives of African Americans. For a sentence to be grammatically complete, it must include both a subject and a main verb. When a sentence lacks either a subject or a main verb, the result is a sentence fragment.In this example all options but (C) are sentence fragments. ● In (A), the phrase “Scenes . . . Americans” is modi- fied by the dependent clause “which ...Tanner,” but there is no main verb. ● In (B), the phrase “Scenes ...Tanner” contains no main verb. ● In (D), the noun “Henry Ossawa Tanner” is modified by “depicting” but is not combined with a main verb. ● And in (E), the noun “Henry Ossawa Tanner” is modified by the dependent clause “whose . . . Americans” but not combined with a main verb. ● (C) is correct. It is the only choice in which a sub- ject (“The paintings of Henry Ossawa Tanner”) is combined with a verb (“depict”) to express a com- plete thought. Correct answer: (C) / Difficulty level: Medium 2. L ooking up from the base of the mountain, the trail seemed more treacherous than it really was. (A) Looking up (B) While looking up (C) By looking up (D) Viewing (E) Viewed When a modifying phrase begins a sentence, it must logi- cally modify the sentence’s subject; otherwise, it is a dan- gling modifier. In this example, every option except (E) is a dangling modifier. ● In (A), the phrase “Looking up from the base of the mountain” does not logically modify the sub- ject “the trail.” A person might stand at the base ofa mountain and look up at a trail, but it is illogical to suggest that a trail looks up from the base of a mountain. ● (B), (C), and (D) are simply variations of the error found in (A). Each results in a sentence that illogi- cally suggests that a trail was looking up from the base of a mountain. ● (E) is correct. Although a trail cannot itself look up from the base of a mountain, a trail can be viewed by someone looking up from the base of a moun- tain, so the phrase “Viewed from the base of the mountain” logically modifies the subject “the trail.” Correct answer: (E) / Difficulty level: Hard Identifying Sentence Errors This question type measures your ability to: ● recognize faults in grammar and usage ● recognize effective sentences that follow the conventions of standard written English Directions The following sentences test your ability to recognize grammar and usage errors. Each sentence contains either a single error or no error at all. No sentence contains more than one error. The error, if there is one, is underlined and lettered. If the sentence con- tains an error, select the one underlined part that must be changed to make the sentence correct. If the sentence is correct, select choice E. In choosing answers, follow the requirements of stan- dard written English. EXAMPLE: T he othe rdelegates and him imme diat ely A B C accepted the resolution dr aft ed b ythe D neutral states. N o e rror E Answering Identifying Sentence Errors Questions Ask yourself if any of the underlined words and phrases in the sentence contains a grammar or usage error. Follow the two outlined steps in answering each Identifying Sentence Errors question. Step 1:Read the entire sentence carefully but quickly, pay- ing attention to underlined choices (A) through (D). In the example above,“The other delegates and him”are the people who “immediately accepted the resolution,” A BC D E

SAT Preparation Booklet 29 and the phrase “drafted by the neutral states”describes “the resolution.”Check each underlined word or phrase for correctness. ● The phrase “The other”correctly modifies the word “delegates.” ● The pronoun “him”is in the wrong case. (One would not say “him”immediately accepted.”) “Him” is an error, but go on to check the other choices, especially if you are not sure. ● The word “immediately,”which modifies the verb “accepted,”is correct. ● The phrase “drafted by”correctly expresses the action of the “neutral states.” Step 2:Select the underlined word or phrase that needs to be changed to make the sentence correct. ● Mark (E) N o e rrorif you believe that the sentence is correct as written. Mark (B) on your answer sheet because the underlined word “him”must be changed to “he”to make the sentence correct. Correct answer: (B) / Difficulty level: Easy Keep in mind that some sentences do not contain an error. Sample Questions 3. The students ha ve disc overedthat the ycan address AB issues more effectively thr oug hletter-writing C campaigns and not through public D demonstrations. N o e rror E ● The error in this sentence occurs at (D). When a comparison is introduced by the adverb “more,” as in “more effectively,”the second part of the com- parison must be introduced by the conjunction “than”rather than “and not.” ● The other options contain no errors. In (A), the plu- ral verb “have discovered”agrees with the plural subject “students.”In (B), the plural pronoun “they”correctly refers to the plural noun “students.” In (C), the preposition “through”appropriately expresses the means by which issues are addressed. ● The sentence may be corrected as follows: The stu- dents have discovered that they can address issues more effectively through letter-writing campaigns thanthrough public demonstrations. Correct answer: (D) / Difficulty level: Easy 4. A fterhours of futile debate, the committee has A decided t o p ost pone further discussion B o f the r esol utionuntil the irnext meeting. CD N o e rror E ● The error in this sentence occurs at (D). A pronoun must agree in number (singular or plural) with the noun to which it refers. Here, the plural pronoun “their”incorrectly refers to the singular noun “committee.” ● The other options contain no errors. In (A), the preposition “A f t e r ”appropriately introduces a phrase that indicates when the committee made its decision. In (B),“to postpone”is the verb form needed to complete the description of the commit- tee’s decision. In (C), the prepositional phrase “of the resolution”appropriately specifies the subject of the postponed discussion. ● The sentence may be corrected as follows: After hours of futile debate, the committee has decided to postpone further discussion of the resolution until its next meeting. Correct answer: (D) / Difficulty level: Medium Improving Paragraphs This type of question measures your ability to: ● edit and revise sentences in the context of a para- graph or entire essay ● organize and develop paragraphs in a coherent and logical manner ● apply the conventions of standard written English Directions Directions:The following passage is an early draft of an essay. Some parts of the passage need to be rewritten. Read the passage and select the best answers for the questions that follow. Some questions are about par- ticular sentences or parts of sentences and ask you to improve sentence structure or word choice. Other questions ask you to consider organization and devel- opment. In choosing answers, follow the require- ments of standard written English. Answering Improving Paragraphs Questions To answer the Improving Paragraphs questions that accompany the draft essay, you will need to note what sen- tences need to be corrected and to know how each of the sentences relate to one another and to the essay as a whole. Follow the outlined steps to answer the questions. Step 1:Read the entire essay quickly to determine its over- all meaning. The essay is intended as a draft, so you will notice errors. Step 2:In answering each question, make sure that your answer about a particular sentence or group of sentences makes sense in the context of the passage as a whole. Choose the best answer from among the choices given, even if you can imagine another correct response.

SAT Preparation Booklet 30 Sample Questions Questions 5–7 are based on the following passage: (1)Many times art history courses focus on the great “masters,” ignoring those women who should have achieved fame.(2)Often women artists like Mary Cassatt have worked in the shadows of their male con- temporaries.(3)They have rarely received much attention during their lifetimes. (4)My art teacher has tried to make up for it by teaching us about women artists and their work.(5)Recently she came to class very excited; she had just read about a little- known artist named Annie Johnson, a high school teacher who had lived all of her life in New Haven, Connecticut.(6)Johnson never sold a painting, and her obituary in 1937 did not even mention her many paint- ings.(7)Thanks to Bruce Blanchard, a Connecticut businessman who bought some of her watercolors at an estate sale. (8)Johnson is finally starting to get the attention that she deserved more than one hundred years ago.(9)Blanchard now owns a private collection of hundreds of Johnson’s works—watercolors, charcoal sketches, and pen-and-ink drawings. (10)There are portraits and there are land- scapes.(11)The thing that makes her work stand out are the portraits.(12)My teacher described them as “unsentimental.” (13)They do not idealize characters. (14)Characters are presented almost photo- graphically.(15)Many of the people in the pictures had an isolated, haunted look. (16)My teacher said that isolation symbol- izes Johnson’s life as an artist. 5. In context, which is the best revision to the under- lined portion of sentence 3 (reproduced below)? The y ha ve rarely received much attention during their lifetimes. (A) In fact, they had (B) Too bad these artists have (C) As a result, these women have (D) In spite of this, women artists (E) Often it is the case that the former have Although sentence 3 is not grammatically wrong, its relationship to the preceding sentence needs to be made clearer. A transitional phrase should be added to emphasize the cause-and-effect relationship between the stated facts—women artists received little attention as a conse- quenceof having worked in the shadows of their male con- temporaries—and the ambiguous pronoun “They”should be replaced with a word or phrase that clearly refers to the“women artists” and not the “male contemporaries” men- tioned in sentence 2. ● (A), (B), and (D) are unsatisfactory because in each case the transitional phrase (“In fact,” “Too bad,” or “In spite of this”) fails to indicate the cause- and-effect relationship. Moreover, both (A) and (B) leave the ambiguity of the pronoun unresolved. ● (E) is unsatisfactory not only because it fails to sig- nal the cause-and-effect relationship but also because it is wordy and illogically combines the adverbs “Often”and “rarely.” ● (C) is correct. The transitional phrase “as a result” clearly indicates a cause-and-effect relationship, and “these women”properly resolves the ambiguity of the pronoun “They.” Correct answer: (C) / Difficulty level: Hard 6. In context, which of the following revisions to sen- tence 7 is most needed? (A) Delete “Thanks to”. (B) Move “Thanks to Bruce Blanchard” to the end of sentence 7. (C) Delete “who”. (D) Change “her” to “Johnson’s”. (E) Change the period to a comma and combine sentence 7 with sentence 8. Sentence 7 is a sentence fragment, with neither a subject nor a main verb to finish the thought it has begun. It says “Thanks to Bruce Blanchard,”but it does not say what happenedthanks to Bruce Blanchard. It should therefore be joined to an independent clause, complete with subject and verb,that indicates what happened as a result of Blanchard’s action. ● (A), (B), and (D) are unsatisfactory because each fails to provide the main verb needed to complete the sen- tence. Each results in another sentence fragment. ● Although (C) results in a complete sentence, the sentence makes little sense in the context of the paragraph because it suggests that Bruce Blanchard is someone other than the Connecticut business- man who bought the watercolors. ● (E) is correct. This change results in a grammati- cally complete sentence that indicates what hap- pened thanks to Bruce Blanchard’s efforts: Johnson began to get the attention she deserved. Correct answer: (E) / Difficulty level: Medium 7.In context, which of the following is the best version of sentence 10 (reproduced below)? There are portraits and there are landscapes. (A) (As it is now) (B) You can see both portraits and landscapes. (C) Therefore, both portraits and landscapes are among her works. (D) Johnson painted both portraits and landscapes. (E)Among them Johnson has portraits and landscapes.

SAT Preparation Booklet 31 In addition to being vague, sentence 10 contains no noun to which the pronoun “her” in sentence 11 may refer. It should be revised so that Johnson is clearly identified as the painter of the portraits and landscapes. ● (A), (B), and (C) are unsatisfactory because each omits any mention of Johnson. ● Though (E) does mention Johnson, it is misleading in that the words “Johnson has”suggest that Johnson is the owner rather than the painter of the portraits and landscapes. ● (D) is correct because it properly identifies Johnson as the painter of the artworks and thus provides an antecedent for the pronoun “her” in sentence 11. Correct answer: (D) / Difficulty level: Easy The Essay The essay will measure your ability to: ● develop a point of view on an issue presented in an excerpt ● use reasoning and evidence based on reading, studies, experience, and observations to support that point of view ● follow the conventions of standard written English Approach to the Essay ● Read the prompt carefully and make sure you write on the topic given. Essays not on topic will receive a zero. ● Decide your viewpoint on the topic. If you have trouble focusing on the main point, try completing this sentence: "When people finish reading my essay, I want them to understand that...." The words you use to fill in that blank might well become part of your thesis statement. ● Spend five minutes on planning. Use your test booklet to create a quick outline. Begin with the thesis from the approach above. You have only twenty-five minutes to write your essay, so don't spend too much time outlining. A little planning time, however, may be essential to make sure that your essay does not wander off topic or stray from your focused thesis statement. ● Vary the sentence structure in your writing. Good writing uses a variety of sentence types to make the writing more interesting while showing the relation- ships between ideas. To give your prose a mature character, vary the sentence structure that commu- nicates how your ideas are related to each other. ● Use clear, precise, and appropriate vocabulary. Appropriate words are accurate and specific, not necessarily long and obscure. ● Leave time to review what you've written. Although you won't have time for full-scale revision, do leave a few minutes for rereading your essay and making minor changes in the wording or even in the struc- ture of what you have written. Directions The essay gives you an opportunity to show how effectively you can develop and express ideas. You should, therefore, take care to develop your point of view, present your ideas logically and clearly, and use language precisely. Your essay must be written on the lines provided on your answer sheet—you will receive no other paper on which to write. You will have enough space if you write on every line, avoid wide margins, and keep your handwriting to a reasonable size. Remember that people who are not familiar with your handwrit- ing will read what you write. Try to write or print so that what you are writing is legible to those readers. You have twenty-five minutes to write an essay on the topic assigned below. DO NOT WRITE ON ANOTH- ER TOPIC. AN OFF-TOPIC ESSAY WILL RECEIVE A SCORE OF ZERO. Think carefully about the issue presented in the fol- lowing excerpt and the assignment below. Prompt: A sense of happiness and fulfillment, not personal gain, is the best motivation and reward for one’s achievements. Expecting a reward of wealth or recognition for achiev- ing a goal can lead to disappointment and frustration. If we want to be happy in what we do in life, we should not seek achieve- ment for the sake of winning wealth and fame. The personal satisfaction of a job well done is its own reward. Assignment:Are people motivated to achieve by personal satisfaction rather than by money or fame? Plan and write an essay in which you develop your point of view on this issue. Support your position with reasoning and examples taken from your reading, studies, experience, or observations. To r e c e i ve an automated score for your response to this essay question, check out The Official SAT Online Course at www.collegeboard.com/satonlinecourse. Sample Essays Essays are scored by experienced high school teachers and college faculty members who primarily teach English, com- position, or language arts courses, or who teach a course in another subject that requires a substantial amount of writing. Each essay will be scored independently by two qualified readers on a scale of 1 to 6, with 6 being the highest score. The combined score for both readers will range from 2 to 12. If the two readers’ scores are more than one point apart, a third reader resolves the discrepancy.

SAT Preparation Booklet 32 Essay #1: This essay received a score of 6 Even though we live in a capitalist society, I still cannot help but believe, despite my own cynicism, that people are more motivated to achieve something for personal satisfac- tion rather than monetary gains. Look at Chekov's short story, “The Bet.” A man agrees to sacrifice fifteen years of his life in prison in exchange for a million dollars. Obviously his motivation for such an extreme bet is wealth, but by the end of the prison sentence, the man could care less about the money. After years of introspec- tion, of reading Shakespeare, The Bible, and textbooks, the man actually comes to despise the money he once sought; the money he signed away fifteen years of his life for. He does not collect his money from the banker, he runs away to be on his own and continue to live the life of solitude he has learned to love, free of money and possessions. Also, in a psychology class, one of the first things students study when they come to the topic of motivation, is exter- nal stimulus versus personal drive. Any textbook will tell one that studies show that a child is more likely to put as much energy as possible into completing a task when it is something that makes him happy, than if he was doing it for a physical reward. A child is more likely to get good grades, if it makes him feel good about himself, than if his parents offer to pay him every time he makes the honor roll. I agree with this theory on motivation because I see it play out everyday in my life. If my older sister had been concerned with money and fame, which reality television tells us every night is important, she would have gone to college after graduating high school. She knew though, that school and learning did not make her happy, and she was not going to suffer through four more years of school just because a college degree could lead to a more successful job. Right now she does not make as much money at her job, but she likes her life and the way she lives; she has more fun answering phones and dealing with other people at work than she would behind a desk in a classroom. This past year I myself have been forced to look at my priorities as well. I have worked hard in school all my life and have made honor roll semester after semester, because I enjoy it. I have not filled up my schedule with classes I did not want because calculus and economics look good on a college transcript. I had a high enough GPA to join the National Honor Society, but I chose not to join because even though it might have impressed some admissions officers, it was not something that was going to make me happy. Instead I spend my time studying Creative Writing, Art History, and the other subjects I feel truly passionate about. There is a pleasure principle in psychology, which basically means that one will do whatever will make them most happy or least unhappy. I think that is true, and I feel that the happiness most people seek out is not about money or luxury. Maybe it looks like that from the media, because advertising says that people want to be like Donald Trump, but that is not real life. Real life is my next door neighbor who gardens as a second job for small fees because he loves to be outside, working with his hands in the nice weather. I am sure no one would mind winning the lottery, but to say that it is our primary motivator in life is sad and untrue. A person who is happy and making minimum wage is likelyto live longer than someone who spends his or her life working sixty four hour weeks at a stressful job to make money hand over fist. Are some people very driven by money? Yes. Is that more important than the personal sat- isfaction that comes from doing something good? Literature, psychology, and our personal lives tell us no, and I hope it stays that way. Why Essay #1 Received a Score of 6 This outstanding essay insightfully and effectively develops the point of view that, “Even though we live in a capitalist society, I still cannot help but believe... that people are more motivated to achieve something for personal satisfaction rather than monetary gains.” The writer demonstrates out- standing critical thinking by focusing on clearly appropriate examples from “Literature, psychology, and our personal lives” to support this position. The essay begins by describ- ing Chekhov’s “The Bet” as a short story that, through its main character's changed priorities after "years of intro- spection" in prison, ultimately places higher value on a “life of solitude... free of money and possessions” than on wealth. The writer continues to demonstrate outstanding critical thinking by offering as evidence the psychological principle of “external stimulus versus personal drive” and several examples of how this principle has been borne out in a sister’s and the writer’s own lives. The essay concludes by once again drawing on psychology, this time the concept of the “pleasure principle,” to reinforce the idea that “the happiness most people seek out is not about money or lux- ury” but lies in “the personal satisfaction that comes from doing something good.” This well organized and clearly focused essay demonstrates coherence and progression of ideas. The essay consistently exhibits skillful use of language and demonstrates meaningful variety in sentence structure (“After years of introspection, of reading Shakespeare, The Bible, and textbooks, the man actually comes to despise the money he once sought; the money he signed away fifteen years of his life for”). Thus, this essay demonstrates clear and consistent mastery and receives a score of 6. Essay#2: This essay received a score of 5 I believe that personal satisfaction, through giving, and providing your best effort, has a greater and more reward- ing outcome over gaining wealth or fame. This is evident through many public examples of people who are known to our society as “celebrities” stooping to a standard of drug reliance and materialism. It seems to me that people who have achieved the money and achieved the fame are never quite satisfied with themselves or the situations they are in. Although many people are based on financial stature, there are also many groups or individuals that are based on goals more helpful to our society as a whole. People who are involved in organizations such as the make-a-wish foundation and Habitat for Humanity are working towards a common goal of giving others opportu- nities, and chances they wouldn’t normally have. These foundations are non-profit, and work with and around our communities to help advance and improve the lives of others. People who do these projects and strive for the betterment

SAT Preparation Booklet 33 of others are not doing it for themselves, for the money, or for the recognition. They are doing it whole heartedly without complaint. To me, people who don’t think of what benefits or rewards they will reap are truely the ones obtaining the most right- eous self-achievement. Through their actions and beleifs, I undoubtedly beleive that these people are gaining the hapi- ness and fulfillment that even the richest or most famous people wish they had. When you go into a project or any type of action not expecting anything back, the things you do will be paid back to you ten-fold in the grand scheme of things. Doing that action, or job, or project, whatever it may be, to the best of your ability will give you a sense of accomplishment like no other because of the amount of effort and giving you put forth. It is too bad people or groups like this aren’t recognized as much as a person who wins a game show or is in a movie. Why Essay #2 Received a Score of 5 This focused essay demonstrates strong critical thinking and effectively develops its point of view (“I believe that personal satisfaction, through giving, and providing your best effort, has a greater and more rewarding outcome over gaining wealth or fame”) by offering a well organized progression of ideas detailing the motivations of and benefits for groups such as “the make-a-wish foundation and Habitat for Humanity,” who “help advance and improve the lives of oth- ers.” Additional focus is provided at the essay's opening and closing by comparing these groups to “many public exam- ples of... ‘celebrities’ stooping to a standard of drug reliance and materialism” because they “are never quite satisfied with themselves or the situation they are in.” The response also exhibits facility in the use of language and variety in sen- tence structure (“People who do these projects and strive for the betterment of others are not doing it for themselves, for the money, or for the recognition. They are doing it whole heartedly without complaint”). Some lapses in the specificity of support prevent the response from earning a higher score. Therefore, to merit a score of 6, this essay needs to provide additional detailed evidence to more evenly and insightfully develop the point of view. Overall, this response exhibits reasonably consistent mastery and receives a score of 5. Essay #3: This essay received a score of 4 Some people in the world today feel that they need to achieve a goal in order to receive fame and money. While others achieve a goal for their own satisfaction. I personally feel that you should achieve a goal for your own satisfac- tion and not for that of fame. The most important thing about someone’s life is to have a positive self-esteem. People need to not worry so much about what others think and just need to start concentrat- ing on doing everything for themselves. Within my life I set a lot of goals from which I hope to achieve. One major goal is that of trying to drop my fifty meter freestyle time to twenty-five seconds. I work day in and day out on this and want to show myself I can do it. I don’t care what oth- ers think about my goals. I want to do this for myself and not for fame or money.A person should not feel the need for fame or money because if they achieve a lifetime goal they should just be happy with themselves. Think about it I mean, how long does money last? Is fame really worth anything? I don’t think so. I mean, I know it would be nice to be recognized for doing something special or extraordinary but truthfully it won’t help you. Fame may make your day or make you smile for a couple of days but when you really think about it, it won’t make you happy for the rest of your life. Yet, if you do something for yourself and keep doing things for yourself you should always be happy with yourself. Remember fame and money don’t last forever but one’s own happiness can last a lifetime and eternity. Therefore it is very obvious that the choice that should be made when trying to find motivation for a goal would be that of per- sonal satisfaction rather than that of fame or money. Why Essay #3 Received a Score of 4 This essay develops the point of view that “you should achieve a goal for your own satisfaction and not for that of fame.” The writer demonstrates competent critical thinking by presenting adequate evidence in a manner that demon- strates some progression of ideas from the short-term gain of “positive self-esteem,” supported by a specific example from the writer's life (“One major goal is that of trying to drop my fifty meter freestyle time”), to the less specifically supported attainment of a “lifetime goal” with greater last- ing value than money or fame. Facility in the use of lan- guage is evident (“Fame may make your day or make you smile for a couple of days but when you really think about it, it won't make you happy for the rest of your life”) but inconsistent (“Yet, if you do something for yourself and keep doing things for yourself you should always be happy with yourself ”). The essay exhibits some variety in sen- tence structure (“A person should not feel the need for fame or money because if they achieve a lifetime goal they should just be happy with themselves. Think about it I mean how long does money last? Is fame really worth any- thing?”). To merit a higher score, the essay needs to devel- op the point of view more insightfully and evenly by pro- viding further detailed evidence as support. This essay demonstrates adequate mastery and receives a score of 4. Essay #4: This essay received a score of 3 Money and fame are nice but if you aren’t satisfied with your personal goals and acheivement, then the money and fame don’t matter. Material things do not matter in the gist of life. People who are rich in one century are forgotten in the next. Mother Teresa accomplished more in a few years than most people do in a lifetime, even though she was not wealthy. If you are passionate for a cause, then the money and fame do not matter. Who remembers who the richest person in the world was in 1792? No one. However, I will always remember that in 1921, Susan B. Anthony fought for the right for a woman to vote. She was not a wealthy woman but had enough passion about her cause that the rewards did not matter more than the accomplishment of her purpose.

SAT Preparation Booklet 34 Why Essay #4 Received a Score of 3 This essay demonstrates developing mastery and some criti- cal thinking by attempting to support the position that “Money and fame are nice but if you aren’t satisfied with your personal goals and acheivement, then the money and fame don't matter.” The writer limits focus to a central rea- son, “People who are rich in one century are forgotten in the next,” and supports this reason using two examples that are specific, though undeveloped and therefore inadequate (“Mother Teresa accomplished more in a few years than most people do in a lifetime.... I will always remember that in 1921, Susan B. Anthony fought for the right for a woman to vote”). Although the writer demonstrates developing facility in the use of language (“Who remembers who the richest person in the world was in 1792? No one. However, I will always remember that in 1921, Susan B. Anthony fought for the right for a woman to vote”), vocabulary is sometimes weak and repetitive (“then the money and fame don’t matter...then the money and fame do not matter...that the rewards did not matter”). To merit a higher score, this essay needs to exhibit stronger critical thinking by provid- ing further focused and detailed evidence to develop the point of view more effectively. Overall, this essay remains in the inadequate category, earning a score of 3. Essay #5: This essay received a score of 2 I have several ideas why it is personal satisfaction and not money or fame that urges people to succeed. My belief is after you succeed in something you do you feel good about yourself, you feel as you just successfully competed your mission that you made for yourself and it makes you feel good inside for example after an actor wins an academy award I think he does not think about money or fame but instead he or she feels as she is great at her job and it makes that person complete and happy inside. Another example are authors who usually never get famous or wealthy when they are alive but its rather the personal feel- ing of fulfillment of happiness that inspires them to write great stories. Those are several reasons why I think its per- sonal feeling and satisfaction and not fame and fortune that inspires people to succeed. Why Essay #5 Received a Score of 2 This essay offers a seriously limited point of view (“its per- sonal feeling and satisfaction and not fame and fortune that inspires people to succeed”) and demonstrates some critical thinking by supporting this position with several brief examples (“after an actor wins an academy award...authors who usually never get famous or wealthy”) of situations in which “after you succeed in something you do you feel good about yourself.” However, this evidence consists of general statements that are insufficient to support the writer’s posi- tion (“I think he does not think about money or fame but instead he or she feels as she is great at her job and it makes that person complete and happy inside”). The response dis- plays very little facility with language due to limited and repetitive vocabulary (“you feel good about yourself...you feel good inside...”). Consequently, to receive a higher score, this essay needs to exhibit more skillful facility in the use oflanguage as well as demonstrate stronger critical thinking by providing additional focused and specific evidence that will adequately develop the point of view. This response demon- strates little mastery and remains at the 2 score point. Essay #6: This essay received a score of 1 My view of the idea that it is personal satisfaction rather than money or fame that motivates people to achieve is sometimes wrong because in sports some people do it for personal satisfaction because they love the game and some people do it for the money because it pays well. For exam- ple, in the NFL there are two types of people, one plays for the check and the other plays for the ring. Why Essay #6 received a Score of 1 Offering little evidence to support the writer's point of view (“My view of the idea that it is personal satisfaction rather than money or fame that motivates people to achieve is sometimes wrong”), this response is fundamentally lacking. The sparse supporting evidence provided is weak (“in sports some people do it for personal satisfaction because they love the game and some people do it for the money”) and repeti- tive (“For example, in the NFL there are two types of people, one plays for the check and the other plays for the ring”). The essay consists of only two sentences, and one of these sentences displays flawed structure (“My view of the idea that it is personal satisfaction rather than money or fame that motivates people to achieve is sometimes wrong because in sports some people do it for personal satisfaction because they love the game and some people do it for the money because it pays well”). To merit a higher score, this essay needs to exhibit more control over sentence structure as well as demonstrate stronger critical thinking by provid- ing further focused and detailed evidence to adequately develop the point of view. Overall, this essay demonstrates very little mastery and is scored a 1. To see additional sample essays, visit the SAT Preparation Center at www.collegeboard.com/srp. Scoring the Essay Essays will be scored in a manner that is fair and consis- tent, using a holistic approach. In holistic scoring, a piece of writing is considered as a total work, the whole of which is greater than the sum of its parts. Even with some errors in spelling, punctuation, and grammar, a student can get a top score on the essay. The highly trained high school and college teachers who score the essays will follow the scoring guide below that focuses on content, organization, language usage, and sen- tence structure. The scoring guide illustrates the importance of directly relevant responses that cannot be coached or memorized ahead of test time. The new SAT essay will nei- ther reward nor punish formulaic approaches to writing, such as the five-paragraph essay. Readers will be trained to recognize and reward a wide variety of essays at each score point.

SAT Preparation Booklet 35 SCORE OF 6 An essay in this category is outstand- ing,demonstrating clear and consis- tent mastery,although it may have a few minor errors. A typical essay: ● effectively and insightfully develops a point of view on the issue and demonstrates outstanding critical thinking, using clearly appropriate examples, reasons, and other evidence to support its position ● is well organized and clearly focused, demonstrating clear coherence and smooth pro- gression of ideas ● exhibits skillful use of lan- guage, using a varied, accu- rate,and apt vocabulary ● demonstrates meaningful variety in sentence structure ● is free of most errors in grammar, usage, and mechanics SCORE OF 5 An essay in this category is effective, demonstrating reasonably consistent mastery,although it will have occa- sional errors or lapses in quality. A typical essay: ● effectively develops a point of view on the issue and demonstrates strong critical thinking, generally using appropriate examples, reasons, and other evidence to support its position ● is well organized and focused, demonstrating coherence and progression of ideas ● exhibits facility in the use of language, using appro- priate vocabulary ● demonstrates variety in sentence structure ● is generally free of most errors in grammar, usage, and mechanics SCORE OF 4 An essay in this category is compe- tent,demonstrating adequate mas- tery,although it will have lapses in quality. A typical essay: ● develops a point of view on the issue and demonstrates competent critical thinking, using adequate examples, reasons, and other evidence to support its position ● is generally organized and focused, demonstrating some coherence and progression of ideas ● exhibits adequate but inconsistent facility in the use of language, using generally appropriate vocabulary ● demonstrates some variety in sentence structure ● has some errors in gram- mar, usage, and mechanics SCORE OF 3 An essay in this category is inade- quate,but demonstrates developing mastery,and is marked by ONE OR MORE of the following weaknesses: ● develops a point of view on the issue, demonstrating some critical thinking, but may do so inconsistently or use inadequate examples, reasons, or other evidence to support its position ● is limited in its organization or focus, or may demon- strate some lapses in coher- ence or progression of ideas ● displays developing facility in the use of language, but sometimes uses weak vocabulary or inappropriate word choice ● lacks variety or demon- strates problems in sentence structure ● contains an accumulation of errors in grammar, usage, and mechanics SCORE OF 2 An essay in this category is seriously limited,demonstrating little mastery,and is flawed by ONE OR MORE of the following weaknesses: ● develops a point of view on the issue that is vague or seriously limited, and demonstrates weak critical thinking, providing inappro- priate or insufficient exam- ples, reasons, or other evi- dence to support its position ● is poorly organized and/or focused, or demonstrates serious problems with coherence or progression of ideas ● displays very little facility in the use of language, using very limited vocabulary or incorrect word choice ● demonstrates frequent problems in sentence structure ● contains errors in grammar, usage, and mechanics so serious that meaning is somewhat obscured SCORE OF 1 An essay in this category is funda- mentally lacking,demonstrating very littleor no mastery,and is severely flawed by ONE OR MORE of the following weaknesses: ● develops no viable point of view on the issue, or provides little or no evi- dence to support its position ● is disorganized or unfo- cused, resulting in a dis- jointed or incoherent essay ● displays fundamental errors in vocabulary ● demonstrates severe flaws in sentence structure ● contains pervasive errors in grammar, usage, or mechanics that persistently interfere with meaning Essays not written on the essay assignment will receive a score of zero. SAT SCORING GUIDE

SAT Preparation Booklet 36Approaches to the Practice Test ● Set aside 3 hours and 20 minutes of uninterrupted time. That way you can complete the entire test in one sitting.Note:the total testing time is 3 hours and 45 minutes, but you save 25 minutes because the unscored section from this practice test was omitted. ● Sit at a desk or table cleared of any other papers or books. You won’t be able to take a dictionary, books, notes, or scratch paper into the test room. ● Allow yourself the specified amount of time for each section. Pace yourself by using a watch (with- out an audible alarm), which is what you are allowed on test day. ● Have a calculator at hand when you take the math sections. This will help you determine how much to use a calculator the day of the test. Use a calcula- tor with which you are familiar. ● Read the test instructions carefully. They are reprinted from the back cover of the test book. On test day, you will be asked to read them before you begin answering questions. ● After you finish the test, read page 84. You can enter your answers to the practice test and receive a scores and skills report, as well as explanations for the questions on the test. You can also receive an automated essay score for your response to the essay question, and view sample essays with explanations. Check out The Official SAT Online Course at www.collegeboard.com/satonlinecourse. Practice SAT About the Practice Test Take the practice test, which starts on page 45, to reinforce your test-taking skills and to be more comfortable when you take the SAT. This practice test will give you a good idea of what to expect on the actual test. However, the test you eventually take will differ in some ways. It may, for example, contain a different number of reading passages, and its sections may be in a different order. Also, this practice SAT includes only nine of the ten sections that the actual test contains.Section 3 is an unscored sec- tion and has been omitted on this test because it contains questions that may be used in future editions of the SAT. The practice test will help you most if you take it under conditions as close as possible to those of the actual test. Finding Your Scores Your raw test scores are placed on the College Board scale of 200 to 800. Use the table on page 85 to find the scaled scores that correspond to your raw scores on this edition of the SAT. The essay will be scored on a scale of 1 to 6, and reported on a scale of 2 to 12. Use the Essay Scoring Guide on page 35 to determine how your particular essay might be scored. You can double the score to get an approximate essay score. Reviewing Your Performance After you score your practice test, analyze your perform- ance. Asking yourself these questions and following the suggestions can help you improve your scores: ● Did you run out of time before you finished a sec- tion?Try to pace yourself so you will have time to answer all the questions you can. Don’t spend too much time on any one question. ● Did you hurry and make careless mistakes?Yo u may have misread the question, neglected to notice the word “except” or “best,” or solved for the wrong value. ● Did you spend too much time reading directions? You should be familiar with the test directions so you don’t have to spend as much time reading them when you take the actual test.

SAT Preparation Booklet 37 PLEASE DO NOT WRITE IN THIS AREA SERIAL # SAT Reasoning Test TM Use a No. 2 pencil only. Be sure each mark is dark and completely fills the intended circle. Completely erase any errors or stray marks. (Print) 1 LastFirst M.I. Your Name: I agree to the conditions on the back of the SAT ® test book. Signature: Home Address: Date Center:(Print)City State Zip Code City StateNumber and Street Female Male SEX 5 DATE OF BIRTH Jan Feb Mar Apr May Jun Jul Aug Sep Oct Nov Dec MONTH DAY YEAR 1 2 3 0 1 2 3 4 5 6 5 6 5 6 7 8 9 0 8 1 2 3 4 7 8 9 0 9 4 7 0 1 2 3 4 5 6 7 8 9 0 1 2 3 4 5 6 7 8 9 0 1 2 3 4 5 6 7 8 9 0 1 2 3 4 5 6 7 8 9 0 1 2 3 4 5 6 7 8 9 0 1 2 3 4 5 6 7 8 9 0 1 2 3 4 5 6 7 8 9 0 1 2 3 4 5 6 7 8 9 0 1 2 3 4 5 6 7 8 9 0 1 2 3 4 5 6 7 8 9 REGISTRATION NUMBER 6 (Copy from Admission Ticket.) 0 1 2 3 4 5 6 7 8 9 0 1 2 3 4 5 6 7 8 9 0 1 2 3 4 5 6 7 8 9 0 1 2 3 4 5 6 7 8 9 0 1 2 3 4 5 6 7 8 9 0 1 2 3 4 5 6 7 8 9 TEST BOOK SERIAL NUMBER 10 (Copy from front of test book.) 0 1 2 3 4 5 6 7 8 9 0 1 2 3 4 5 6 7 8 9 0 1 2 3 4 5 6 7 8 9 0 1 2 3 4 5 6 7 8 9 0 1 2 3 4 5 6 7 8 9 TEST CENTER 7 (Supplied by Test Center Supervisor.) TEST FORM (Copy from back of test book.) 9 Important: Fill in items 8 and 9 exactly as shown on the back of test book. 0 1 2 3 4 5 6 7 8 9 0 1 2 3 4 5 6 7 8 9 0 1 2 3 4 5 6 7 8 9 0 1 2 3 4 5 6 7 8 9 0 1 2 3 4 5 6 7 8 9 0 1 2 3 4 5 6 7 8 9 0 1 2 3 4 5 6 7 8 9 0 1 2 3 4 5 6 7 8 9 0 1 2 3 4 5 6 7 8 9 SOCIAL SECURITY NUMBER 3 FOR OFFICIAL USE ONLY 0 1 2 3 4 5 6 7 8 9 0 1 2 3 4 5 6 7 8 9 0 1 2 3 4 5 6 7 8 9 0 1 2 3 4 5 6 7 8 9 Last Name (First 4 Letters) YOUR NAME 2 A B C D E F G H I J K L M N O P Q R S T U V W X Y Z A B C D E F G H I J K L M N O P Q R S T U V W X Y Z A B C D E F G H I J K L M N O P Q R S T U V W X Y Z A B C D E F G H I J K L M N O P Q R S T U V W X Y Z A B C D E F G H I J K L M N O P Q R S T U V W X Y Z A B C D E F G H I J K L M N O P Q R S T U V W X Y Z First Init.Mid. Init. – ’ – ’ – ’ Copyright © 2004 by College Entrance Examination Board. All rights reserved. College Board, SAT, and the acorn logo are registered trademarks of the College Entrance Examination Board. SAT Reasoning Test and SAT Subject Tests are trademarks owned by the College Entrance Examination Board. FORM CODE 1 2 3 4 5 6 7 8 9 0 (Copy and grid as on back of test book.) A C H I J K O L M P B F G Q S V E D R N U T W Z Y X A C H I J K O L M P B F G Q S V E D R N U T W Z Y X A C H I J K O L M P B F G Q S V E D R N U T W Z Y X A C H I J K O L M P B F G Q S V E D R N U T W Z Y X 1 2 3 4 5 6 7 8 9 0 1 2 3 4 5 6 7 8 9 0 8 170232-001:654321 ISD515011030-36390 • NS114E1800 • Printed in U.S.A. 724843

SAT Preparation Booklet 38 Ye s Page 2 I grant the College Board the unlimited right to use, reproduce, and publish my essay for any and all purposes. My name will not be used in any way in conjunction with my essay. I understand that I am free to mark "No," with no effect on my score. No Begin your essay on this page. If you need more space, continue on the next page. Do not write outside of the essay box. Continue on the next page if necessary. SECTION 1

SAT Preparation Booklet 39 PLEASE DO NOT WRITE IN THIS AREA SERIAL # Page 3 Continuation of ESSAY Section 1 from previous page. Write below only if you need more space.

SAT Preparation Booklet 40 9 31 32 33 34 35 36 37 38 39 40 A B C D E A B C D E A B C D E A B C D E A B C D E A B C D E A B C D E A B C D E A B C D E A B C D E 21 22 23 24 25 26 27 28 29 30 A B C D E A B C D E A B C D E A B C D E A B C D E A B C D E A B C D E A B C D E A B C D E A B C D E 11 12 13 14 15 16 17 18 19 20 A B C D E A B C D E A B C D E A B C D E A B C D E A B C D E A B C D E A B C D E A B C D E A B C D E 1 2 3 4 5 6 7 8 9 10 A B C D E A B C D E A B C D E A B C D E A B C D E A B C D E A B C D E A B C D E A B C D E A B C D E 31 32 33 34 35 36 37 38 39 40 A B C D E A B C D E A B C D E A B C D E A B C D E A B C D E A B C D E A B C D E A B C D E A B C D E 21 22 23 24 25 26 27 28 29 30 A B C D E A B C D E A B C D E A B C D E A B C D E A B C D E A B C D E A B C D E A B C D E A B C D E 11 12 13 14 15 16 17 18 19 20 A B C D E A B C D E A B C D E A B C D E A B C D E A B C D E A B C D E A B C D E A B C D E A B C D E 1 2 3 4 5 6 7 8 9 10 A B C D E A B C D E A B C D E A B C D E A B C D E A B C D E A B C D E A B C D E A B C D E A B C D E 1410 1511 1612 1713 18 SECTION 2 SECTION 3 Use the answer spaces in the grids below for Section 2 or Section 3 only if you are told to do so in your test book. CAUTION Student-Produced Responses ONLY ANSWERS ENTERED IN THE CIRCLES IN EACH GRID WILL BE SCORED. YOU WILL NOT RECEIVE CREDIT FOR ANYTHING WRITTEN IN THE BOXES ABOVE THE CIRCLES. Page 4 Start with number 1 for each new section. If a section has fewer questions than answer spaces, leave the extra answer spaces blank. Be sure to erase any errors or stray marks completely. 0 0 0 1 2 3 4 5 6 7 8 9 1 2 3 4 5 6 7 8 9 1 2 3 4 5 6 7 8 9 1 2 3 4 5 6 7 8 9 ∕ ∕ . . . . 0 0 0 1 2 3 4 5 6 7 8 9 1 2 3 4 5 6 7 8 9 1 2 3 4 5 6 7 8 9 1 2 3 4 5 6 7 8 9 ∕ ∕ . . . . 0 0 0 1 2 3 4 5 6 7 8 9 1 2 3 4 5 6 7 8 9 1 2 3 4 5 6 7 8 9 1 2 3 4 5 6 7 8 9 ∕ ∕ . . . . 0 0 0 1 2 3 4 5 6 7 8 9 1 2 3 4 5 6 7 8 9 1 2 3 4 5 6 7 8 9 1 2 3 4 5 6 7 8 9 ∕ ∕ . . . . 0 0 0 1 2 3 4 5 6 7 8 9 1 2 3 4 5 6 7 8 9 1 2 3 4 5 6 7 8 9 1 2 3 4 5 6 7 8 9 ∕ ∕ . . . . 0 0 0 1 2 3 4 5 6 7 8 9 1 2 3 4 5 6 7 8 9 1 2 3 4 5 6 7 8 9 1 2 3 4 5 6 7 8 9 ∕ ∕ . . . . 0 0 0 1 2 3 4 5 6 7 8 9 1 2 3 4 5 6 7 8 9 1 2 3 4 5 6 7 8 9 1 2 3 4 5 6 7 8 9 ∕ ∕ . . . . 0 0 0 1 2 3 4 5 6 7 8 9 1 2 3 4 5 6 7 8 9 1 2 3 4 5 6 7 8 9 1 2 3 4 5 6 7 8 9 ∕ ∕ . . . . 0 0 0 1 2 3 4 5 6 7 8 9 1 2 3 4 5 6 7 8 9 1 2 3 4 5 6 7 8 9 1 2 3 4 5 6 7 8 9 ∕ ∕ . . . . 0 0 0 1 2 3 4 5 6 7 8 9 1 2 3 4 5 6 7 8 9 1 2 3 4 5 6 7 8 9 1 2 3 4 5 6 7 8 9 ∕ ∕ . . . . Section 3, the equating section of this practice test, has been omitted.

SAT Preparation Booklet 41 9 31 32 33 34 35 36 37 38 39 40 A B C D E A B C D E A B C D E A B C D E A B C D E A B C D E A B C D E A B C D E A B C D E A B C D E 21 22 23 24 25 26 27 28 29 30 A B C D E A B C D E A B C D E A B C D E A B C D E A B C D E A B C D E A B C D E A B C D E A B C D E 11 12 13 14 15 16 17 18 19 20 A B C D E A B C D E A B C D E A B C D E A B C D E A B C D E A B C D E A B C D E A B C D E A B C D E 1 2 3 4 5 6 7 8 9 10 A B C D E A B C D E A B C D E A B C D E A B C D E A B C D E A B C D E A B C D E A B C D E A B C D E 31 32 33 34 35 36 37 38 39 40 A B C D E A B C D E A B C D E A B C D E A B C D E A B C D E A B C D E A B C D E A B C D E A B C D E 21 22 23 24 25 26 27 28 29 30 A B C D E A B C D E A B C D E A B C D E A B C D E A B C D E A B C D E A B C D E A B C D E A B C D E 11 12 13 14 15 16 17 18 19 20 A B C D E A B C D E A B C D E A B C D E A B C D E A B C D E A B C D E A B C D E A B C D E A B C D E 1 2 3 4 5 6 7 8 9 10 A B C D E A B C D E A B C D E A B C D E A B C D E A B C D E A B C D E A B C D E A B C D E A B C D E 1410 1511 1612 1713 18 Use the answer spaces in the grids below for Section 4 or Section 5 only if you are told to do so in your test book. CAUTION Student-Produced Responses Page 5 Start with number 1 for each new section. If a section has fewer questions than answer spaces, leave the extra answer spaces blank. Be sure to erase any errors or stray marks completely. SECTION 4 SECTION 5 ONLY ANSWERS ENTERED IN THE CIRCLES IN EACH GRID WILL BE SCORED. YOU WILL NOT RECEIVE CREDIT FOR ANYTHING WRITTEN IN THE BOXES ABOVE THE CIRCLES. 0 0 0 1 2 3 4 5 6 7 8 9 1 2 3 4 5 6 7 8 9 1 2 3 4 5 6 7 8 9 1 2 3 4 5 6 7 8 9 ∕ ∕ . . . . 0 0 0 1 2 3 4 5 6 7 8 9 1 2 3 4 5 6 7 8 9 1 2 3 4 5 6 7 8 9 1 2 3 4 5 6 7 8 9 ∕ ∕ . . . . 0 0 0 1 2 3 4 5 6 7 8 9 1 2 3 4 5 6 7 8 9 1 2 3 4 5 6 7 8 9 1 2 3 4 5 6 7 8 9 ∕ ∕ . . . . 0 0 0 1 2 3 4 5 6 7 8 9 1 2 3 4 5 6 7 8 9 1 2 3 4 5 6 7 8 9 1 2 3 4 5 6 7 8 9 ∕ ∕ . . . . 0 0 0 1 2 3 4 5 6 7 8 9 1 2 3 4 5 6 7 8 9 1 2 3 4 5 6 7 8 9 1 2 3 4 5 6 7 8 9 ∕ ∕ . . . . 0 0 0 1 2 3 4 5 6 7 8 9 1 2 3 4 5 6 7 8 9 1 2 3 4 5 6 7 8 9 1 2 3 4 5 6 7 8 9 ∕ ∕ . . . . 0 0 0 1 2 3 4 5 6 7 8 9 1 2 3 4 5 6 7 8 9 1 2 3 4 5 6 7 8 9 1 2 3 4 5 6 7 8 9 ∕ ∕ . . . . 0 0 0 1 2 3 4 5 6 7 8 9 1 2 3 4 5 6 7 8 9 1 2 3 4 5 6 7 8 9 1 2 3 4 5 6 7 8 9 ∕ ∕ . . . . 0 0 0 1 2 3 4 5 6 7 8 9 1 2 3 4 5 6 7 8 9 1 2 3 4 5 6 7 8 9 1 2 3 4 5 6 7 8 9 ∕ ∕ . . . . 0 0 0 1 2 3 4 5 6 7 8 9 1 2 3 4 5 6 7 8 9 1 2 3 4 5 6 7 8 9 1 2 3 4 5 6 7 8 9 ∕ ∕ . . . .

SAT Preparation Booklet 42 9 31 32 33 34 35 36 37 38 39 40 A B C D E A B C D E A B C D E A B C D E A B C D E A B C D E A B C D E A B C D E A B C D E A B C D E 21 22 23 24 25 26 27 28 29 30 A B C D E A B C D E A B C D E A B C D E A B C D E A B C D E A B C D E A B C D E A B C D E A B C D E 11 12 13 14 15 16 17 18 19 20 A B C D E A B C D E A B C D E A B C D E A B C D E A B C D E A B C D E A B C D E A B C D E A B C D E 1 2 3 4 5 6 7 8 9 10 A B C D E A B C D E A B C D E A B C D E A B C D E A B C D E A B C D E A B C D E A B C D E A B C D E 31 32 33 34 35 36 37 38 39 40 A B C D E A B C D E A B C D E A B C D E A B C D E A B C D E A B C D E A B C D E A B C D E A B C D E 21 22 23 24 25 26 27 28 29 30 A B C D E A B C D E A B C D E A B C D E A B C D E A B C D E A B C D E A B C D E A B C D E A B C D E 11 12 13 14 15 16 17 18 19 20 A B C D E A B C D E A B C D E A B C D E A B C D E A B C D E A B C D E A B C D E A B C D E A B C D E 1 2 3 4 5 6 7 8 9 10 A B C D E A B C D E A B C D E A B C D E A B C D E A B C D E A B C D E A B C D E A B C D E A B C D E 1410 1511 1612 1713 18 SECTION 6 SECTION 7 Use the answer spaces in the grids below for Section 6 or Section 7 only if you are told to do so in your test book. CAUTION Student-Produced Responses Start with number 1 for each new section. If a section has fewer questions than answer spaces, leave the extra answer spaces blank. Be sure to erase any errors or stray marks completely. ONLY ANSWERS ENTERED IN THE CIRCLES IN EACH GRID WILL BE SCORED. YOU WILL NOT RECEIVE CREDIT FOR ANYTHING WRITTEN IN THE BOXES ABOVE THE CIRCLES. PLEASE DO NOT WRITE IN THIS AREA SERIAL # Page 6 0 0 0 1 2 3 4 5 6 7 8 9 1 2 3 4 5 6 7 8 9 1 2 3 4 5 6 7 8 9 1 2 3 4 5 6 7 8 9 ∕ ∕ . . . . 0 0 0 1 2 3 4 5 6 7 8 9 1 2 3 4 5 6 7 8 9 1 2 3 4 5 6 7 8 9 1 2 3 4 5 6 7 8 9 ∕ ∕ . . . . 0 0 0 1 2 3 4 5 6 7 8 9 1 2 3 4 5 6 7 8 9 1 2 3 4 5 6 7 8 9 1 2 3 4 5 6 7 8 9 ∕ ∕ . . . . 0 0 0 1 2 3 4 5 6 7 8 9 1 2 3 4 5 6 7 8 9 1 2 3 4 5 6 7 8 9 1 2 3 4 5 6 7 8 9 ∕ ∕ . . . . 0 0 0 1 2 3 4 5 6 7 8 9 1 2 3 4 5 6 7 8 9 1 2 3 4 5 6 7 8 9 1 2 3 4 5 6 7 8 9 ∕ ∕ . . . . 0 0 0 1 2 3 4 5 6 7 8 9 1 2 3 4 5 6 7 8 9 1 2 3 4 5 6 7 8 9 1 2 3 4 5 6 7 8 9 ∕ ∕ . . . . 0 0 0 1 2 3 4 5 6 7 8 9 1 2 3 4 5 6 7 8 9 1 2 3 4 5 6 7 8 9 1 2 3 4 5 6 7 8 9 ∕ ∕ . . . . 0 0 0 1 2 3 4 5 6 7 8 9 1 2 3 4 5 6 7 8 9 1 2 3 4 5 6 7 8 9 1 2 3 4 5 6 7 8 9 ∕ ∕ . . . . 0 0 0 1 2 3 4 5 6 7 8 9 1 2 3 4 5 6 7 8 9 1 2 3 4 5 6 7 8 9 1 2 3 4 5 6 7 8 9 ∕ ∕ . . . . 0 0 0 1 2 3 4 5 6 7 8 9 1 2 3 4 5 6 7 8 9 1 2 3 4 5 6 7 8 9 1 2 3 4 5 6 7 8 9 ∕ ∕ . . . .

SAT Preparation Booklet 43 Page 7 Start with number 1 for each new section. If a section has fewer questions than answer spaces, leave the extra answer spaces blank. Be sure to erase any errors or stray marks completely. 31 32 33 34 35 36 37 38 39 40 A B C D E A B C D E A B C D E A B C D E A B C D E A B C D E A B C D E A B C D E A B C D E A B C D E 21 22 23 24 25 26 27 28 29 30 A B C D E A B C D E A B C D E A B C D E A B C D E A B C D E A B C D E A B C D E A B C D E A B C D E 11 12 13 14 15 16 17 18 19 20 A B C D E A B C D E A B C D E A B C D E A B C D E A B C D E A B C D E A B C D E A B C D E A B C D E 1 2 3 4 5 6 7 8 9 10 A B C D E A B C D E A B C D E A B C D E A B C D E A B C D E A B C D E A B C D E A B C D E A B C D E SECTION 8 31 32 33 34 35 36 37 38 39 40 A B C D E A B C D E A B C D E A B C D E A B C D E A B C D E A B C D E A B C D E A B C D E A B C D E 21 22 23 24 25 26 27 28 29 30 A B C D E A B C D E A B C D E A B C D E A B C D E A B C D E A B C D E A B C D E A B C D E A B C D E 11 12 13 14 15 16 17 18 19 20 A B C D E A B C D E A B C D E A B C D E A B C D E A B C D E A B C D E A B C D E A B C D E A B C D E 1 2 3 4 5 6 7 8 9 10 A B C D E A B C D E A B C D E A B C D E A B C D E A B C D E A B C D E A B C D E A B C D E A B C D E SECTION 9 31 32 33 34 35 36 37 38 39 40 A B C D E A B C D E A B C D E A B C D E A B C D E A B C D E A B C D E A B C D E A B C D E A B C D E 21 22 23 24 25 26 27 28 29 30 A B C D E A B C D E A B C D E A B C D E A B C D E A B C D E A B C D E A B C D E A B C D E A B C D E 11 12 13 14 15 16 17 18 19 20 A B C D E A B C D E A B C D E A B C D E A B C D E A B C D E A B C D E A B C D E A B C D E A B C D E 1 2 3 4 5 6 7 8 9 10 A B C D E A B C D E A B C D E A B C D E A B C D E A B C D E A B C D E A B C D E A B C D E A B C D E SECTION 10

Timing • You will have 3 hours and 45 minutes to work on this test.  There are ten separately timed sections:  One 25-minute essay  Six other 25-minute sections  Two 20-minute sections  One 10-minute section  You may work on only one section at a time.  The supervisor will tell you when to begin and end each section.  If you finish a section before time is called, check your work on that section. You may NOT turn to any other section.  Work as rapidly as you can without losing accuracy. Don’t waste time on questions that seem too difficult for you. Marking Answers  Carefully mark only one answer for each question.  Make sure each mark is dark and completely fills the circle.  Do not make any stray marks on your answer sheet.  If you erase, do so completely. Incomplete erasures may be scoredas intended answers.  Use only the answer spaces that correspond to the question numbers.  You may use the test book for scratchwork, but you will not receive credit for anything written there.  After time has been called, you may not transfer answers to your answer sheet or fill in circles.  You may not fold or remove pages or portions of a page from this book, or take the book or answer sheet from the testing room. Scoring  For each correct answer to a question, you receive one point.  For questions you omit, you receive no points.  For a wrong answer to a multiple-choice question, you loseone-fourth of a point.  If you can eliminate one or more of the answer choices as wrong, you increase your chances of choosing the correct answer and earning one point.  If you can’t eliminate any choice, move on. You can return to the question later if there is time.  For a wrong answer to a student-produced response (“grid-in”) ma\ th question, you don’t lose any points.  The essay is scored on a 1 to 6 scale by two different readers. The total essay score is the sum of the two readers’ scores.  An off-topic or blank essay will receive a score of zero. The passages for this test have been adapted from published material. The ideas contained in them do not necessarily represent the opinions of\ the College Board or Educational Testing Service. SAT Reasoning Test — General Directions DO NOT OPEN THIS BOOK UNTIL THE SUPERVISOR TELLS YOU TO DO SO. YOUR NAME (PRINT) LAST FIRSTMI TEST CENTER NUMBER NAME OF TEST CENTER ROOM NUMBER IMPORTANT: The codes below are unique to your test book. Copy them on your answer sheet in boxes 8 and 9 and fill in the corresponding circles exactly as shown. UNAUTHORIZED REPRODUCTION OR USE OF ANY PART OF THIS TEST IS PROHIBITED.\ 725383 A B C D E F G H I J K L M N O P Q RS T U V W X YZ A B C D E F G H I J K L M N O P Q RS T U V W X YZ 0 1 2 3 4 5 6 7 8 9 0 1 2 3 4 5 6 7 8 90 1 2 3 4 5 6 7 8 9 A B C D E F G H I J K L M N O P Q RS T U V W X YZ A B C D E F G H I J K L M N O P Q RS T U V W X YZ FORM CODE 8(Copy and grid as on back of test book.) ABCD1 23 TEST FORM 9 (Copy from back of test book) 4162068 SAT Preparation Booklet 44

ESSAY Time — 25 minutes Turn to page 2 of your answer sheet to write your ESSAY. The essay gives you an opportunity to show how effectively you can develop and express ideas. You should, therefore, take care to develop your point of view, present your ideas logically and clearly, and use language precisely. Your essay must be written on the lines provided on your an swer sheet—you will receive no other paper on which to write. You will have enough space if you write on every line, avoid wide margins, and keep your handwriting to a reasonable size. Remember that people who are not familiar with your handwriting will read what you write. Try to write or print so that what you are writing is legible to those readers. You have twenty-five minutes to write an essay on the topic assigned below. DO NOT WRITE ON ANOTHER TOPIC. AN OFF-TOPIC ESSAY WILL RECEIVE A SCORE OF ZERO. Think carefully about the issue presented in the following excerpt and the assignment below. People who like to think of themselves as tough-minded and realistic tend to take it for granted that human nature is “selfish” and that life is a struggle in which only the fittest may survive. According to this view, the basic law by which people must live is the law of the jungle. The “fittest” are those people who can bring to the struggle superior force, superior cunning, and superior ruthlessness. Adapted from S.I. Hayakawa, Language in Thought and Action Assignment: Do people have to be highly competitive in order to succeed? Plan and write an essay in which you develop your point of view on this issue. S upport your position with reasoning and examples taken from your reading, studies, experience, or observations. DO NOT WRITE YOUR ESSAY IN YOUR TEST B OOK. You will receive credit only for what you write on your answer sheet. BEGIN WRITING YOUR ESSAY ON PAGE 2 OF THE ANSWER SHEET. If you finish before time is called, you may check your work on this section only. Do not turn to any other section in the test. SAT Preparation Booklet 45

SECTION 2 Time — 25 minutes 20 Questions Turn to Section 2 (page 4) of your answer sheet to answer the questions in this section. Directions: For this section, solve each problem and decide which is the best of the choices given. Fill in the corresponding circle on the answer sheet. You may use any available space for scratchwork. 1. If x2 36 0 -= , which of the following could be a value of x ? (A) 6 - (B) 4- (C) 0 (D) 3 (E) 12 2. The length of a rectangular rug is 2 feet more than its width. If the length of the rug is 8 feet, what is the area of the rug in square feet? (A) 16 (B) 48 (C) 66 (D) 80 (E) 96 3. If r t =2 and t = 3, what is the value of 2 r ? (A) 2 (B) 4 (C) 6 (D) 8 (E) 12 Some integers in set X are even. 4. If the statement above is true, which of the following must also be true? (A) If an integer is even, it is in set X. (B) If an integer is odd, it is in set X. (C) All integers in set X are even. (D) All integers in set X are odd. (E) Not all integers in set X are odd. SAT Preparation Booklet 46

5. A triangle has a perimeter of 13 and one side of length 3. If the lengths of the other two sides are equal, what is the length of each of them? (A) 4 (B) 5 (C) 6 (D) 7 (E) 8 6. The grid above shows the number of hours worked last week by 12 students of various ages at after-school jobs. Which of the following is true, according to this grid? (A) Half of the students worked more than 12 hours each. (B) One student worked exactly 15 hours. (C) One 16 year old worked more than 11 hours. (D) More 18 year olds than 17 year olds worked at least 13 hours each. (E) Most of the students were under 16 years old. 7. Squaring the product of z and 5 gives the same result as squaring the sum of z and 5. Which of the following equations could be used to find all possible values of z? (A) 552 2zz =+ af (B) 5 5 2 22zza f =+ (C) 55222zz=+ (D) 5 5 22zza f a f =+ (E) 55222zz =+ 8. If as many 7-inch pieces of wire as possible are cut from a wire that is 3 feet long, what is the total length of the wire that is left over? (12 inches = 1 foot) (A) 1 inch (B) 2 inches (C) 3 inches (D) 4 inches (E) 5 inches SAT Preparation Booklet 47

9. Which of the lettered points in the figure above has coordinates (, )xy such that 5? xy += (A) A (B) B (C) C (D) D (E) E { } { } 45 11 ,1, , 4, , 7 72 2 47 ,,4,7 74 A B = = 10. If n is a member of both set A and set B above, which of the following must be true? I. n is an integer. II. 4 n is an integer. III. n = 4 (A) None (B) II only (C) I and II only (D) I and III only (E) I, II, and III 11. If y is directly proportional to x and if y = 20 when x = 6, what is the value of y when x = 9? (A) 10 3 (B) 40 3 (C) 23 (D) 27 (E) 30 12. The nth term of a sequence is defined to be 4 3. n+ The 50th term is how much greater than the 45th term? (A) 5 (B) 16 (C) 20 (D) 23 (E) 24 SAT Preparation Booklet 48

13. Which of the rectangular solids shown above has a volume closest to the volume of a right circular cylinder with radius 2 and height 4 ? (A) A (B) B (C) C (D) D (E) E jx =− 2 0.49 kx =- 049 2 . af mx =− 3 0.49 14. If x is a negative integer, what is the ordering of j, k, and m from least to greatest? (A) jkm

17. The city library donated some children’s books to Mr. Clark’s first-grade class. If each student takes 4 books, there will be 20 books left. If 3 students do not take a book and the rest of the students take 5 books each, there will be no books left. How many books were donated to the class? (A) 120 (B) 140 (C) 160 (D) 175 (E) 185 18. In the figure above, if line  has a slope of -2, what is the y-intercept of ? (A) 7 (B) 8 (C) 9 (D) 10 (E) 12 19. For all positive integers w and y, where wy > , let the operation ƒ be defined by 2 . 2wy wy wy + - ƒ= For how many positive integers w is 1 wƒ equal to 4 ? (A) None (B) One (C) Two (D) Four (E) More than four 20. The figure above represents four offices that will be assigned randomly to four employees, one employee per office. If Karen and Tina are two of the four employees, what is the probability that each will be assigned an office indicated with an X ? (A) 1 16 (B) 1 12 (C) 1 6 (D) 1 4 (E) 1 2 STOP If you finish before time is called, you may check your work on this section only. Do not turn to any other section in the test. SAT Preparation Booklet 50

. . . . . .